You are on page 1of 43

Prepared for students. Amir Mullick -- YouTube.com/USMLELive Best of luck.

Hematol
ogy
andOncol
ogy


You’
rea
lwa
yss
ome
bod
y’st
ype
!(b
loo
dty
pe,
tha
tis
)” E
mbr
yol
ogy 3
96
—Bl
oodLi
nk
Anat
omy 4
00

Allt
hes
oar
ing
sofmymi
ndb
egi
ninmyb
loo
d.”
—Ra
ine
rMa
riaRi
lk
e Phy
siol
ogy 4
02

Thebe
stb
loo
dwi
lla
tso
met
imeg
eti
ntoaf
oolo
ramo
squi
to.
” Pa
thol
ogy 406
—Aus
ti
nO’
Mal
ley
Phar
mac
ology 427

Whe ns tudyinghe matol


ogy,payc l
o s
ea tt
entiontothema nyc ros
s
connecti
onst oimmuno logy
.Ma kesurey ouma ste
rthedif
fer
entt
ypes
ofanemi as
.Bec omforta
bleint
erpre
tingbloodsme ar
s.Whenrevi
ewing
oncol
o gi
cdr ugs,fo
cuso nme chani
smsa nda dver
seeff
ect
srat
herthan
deta
ilsofcli
nicaluses
,whichma ybeloweryield
.

Ple
asenotetha
tso
lidt
umo
rsa
rec
ove
redi
nthe
irr
espe
cti
veor
gan
sy
ste
mc ha
pte
rs.

All blood cells are made via hematopoiesis. They all derive from the same multipotent stem cell, so stem cells can differentiate
into multiple cell lines but not other tissues. These are called hematopoietic stem cells, which first divide to replace themselves
before differentiating. Then you have myeloid and lymphoid cells.
Prepared for students. Amir Mullick -- YouTube.com/USMLELive Best of luck.
S ectIonI I I He mat ol
ogyandonc ol ogy he matol
ogya
ndonc ol
ogy
—emB
Ryolog y

he
mat
ologyandonc
ology
—emB
Ryol
ogy

F
etal
eryt
hropoi
esi
s Fe
tale
rythropoi
esisoccursin: Yo ungLi v erSy nt he si ze sBl ood .
Yolksac(3–8we eks) Yolk sac stops first, Spleen second, Liver stops third at birth, and the last to stop is the bone marrow.
Liver(6weeks–birt
h)
Spleen(10–28we eks
)
Bonema r r
ow( 18we e
k stoadul t)
Hemogl
obi
n Embryonicglobins:ζandε .zeta and epsilon
de
vel
opment Fet
alhemo globi
n( HbF)=α2γ 2.alpha gamma Fr
om f
eta
ltoadul
themo
glob
in:
Adul
themo globin( HbA1)=α2β 2.alpha beta AlhaAl
p wa
ys;GammaGoes,Be
c sBe
ome ta.
HbFha shigheraffini
tyforO2duet ol essa vid 2,3-Bisphosphoglyceric acid is a 3-carbon isomer of the glycolytic intermediate
bi
ndingof2,3-BPG,a l
lowingHbFt oe x t
r a
ct 1,3-BPG and is present in human RBC. It binds with greater affinity to
deoxygenated Hb (in RBC new respiring tissue) than it does to oxygenated Hb
O2from ma t
e r
na lhemo gl
obin( Hb A1a nd (in the lungs) due to conformational differences. It interacts with deoxygenated
Hb beta subunits and decreases the affinity for oxygen and allosterically
HbA2)acrossthep la
centa.Hb A2( α2δ 2)i sa promotes the release of remaining oxygen molecules bound to the Hb, thus,
f
orm ofadulthe mo g
lobinprese ntins ma ll enhancing the ability of RBC to release oxygen near tissues that need it most.
This makes 2,3-BPG an allosteric effector.
amounts.
B
IRTH

Si
t
eof Y
olk i
e
ryt
hro
poie
si
s s
ac Lver B
onema
rro
w
S
ple
en
5
0 α

4
0
F
eta
l(Hb
F) γ A
dul
t(
HbA1)
%oft
otl3
a 0
g
lob
insy
nth
esi
s
2
0 β
εE
mbr
yon
icg
lob
ins
1
0 ζ

We
eks
:6 1
2 1
8 2
4 3
0 3
6 6 1
2 1
8 2
4 3
0 3
6 4
2 >
>
E
MBR
YO F
ETUS(
wee
ks) P
OST
NAT
AL(
mon
ths
) A
DUL
T>>

Hemoglobin exists in two forms, a taut (tense) form and a relaxed form. Factors such as low pH, high CO2, and high 2,3-bpg
(the right shift) favor the taut form, which has low oxygen affinity and releases oxygen in the tissues. Conversely, left shift
factors favor the relaxed form. Inversely, at low partial pressures (such as those present in respiring tissues), the low affinity
taut state is favored because respiring tissues will need the oxygen so the Hb lets go of it.

Think-- Taut has low oxygen affinity, so it is tensed. Relaxed has high oxygen affinity so it has oxygen and its not tensed.
Prepared for students. Amir Mullick -- YouTube.com/USMLELive Best of luck.
He mat ol ogyandonc ol
ogy hema tol
ogyandonc ol
ogy—e mBR
y ogy S
ol ect IonI I
I
People develop antibodies to surface antigens they do not have due to an immune response to bacteria who have antigens that mimic the A and B antigens. Compatibility is key for blood
transfusions to avoid the type 2 Hypersensitivity; type 2 because reaction occurs to cell surface antigens. So if you have type A blood, you make Anti-B ab, and the converse is true too. If you
have blood type AB, you will make no antibodies so type AB is a universal receiver. If you have type O blood, you make both antibodies so you can only receive type O blood. Since type O blood
have no antigens, type O is the universal donor since no antibodies will react with it. If you have a person with type B blood, and you give them type A blood, the host Anti-A ab will cause a type 2
Bl oodgr oups hypersensitivity reaction (antibody reaction against an antigen on a cell surface) leading to hemolysis.

AB
Ocl
ass
ific
ati
on Rhc
las
sific
ati
on

A B AB O Rh Rh
R
BCt
ype

A B A
B O

Grou
panti
gen
son
RBCsur
fac
e A B A&B Rh(
D)
NONE NONE
So it can be given
to anyone.

A
nti
bod
iesi
npl
asma An
ti
-B An
ti
-A An
ti
-A An
ti
-B An
ti
-D
ABO are IgM, while the
Rh are IgG. IgM do not NONE NONE
cross the placenta, while So it can receive
IgG can cross via the any blood.
FcRn receptor. I
gM I
gM I
gM,
IgG I
gG

Cl
i
nic
alr
ele
van
ce R
ecei
veBorAB R
ecei
veAorA
B Uni
ver
sal
rec
ipi
ent Re
cei
veanynon-O Ca
nre
cei
vee
it
her Treatmoth
erwit
h
hemol
yt
ic hemol
yt
ic ofR
BCs;uni
ver
sal hemoly
ti
c R⊕o
h rR
h⊝b l
ood a
nti
-DIgGdur
ingand

r
eac
ti
on r
eac
ti
on dono
rofpl
asma r
eact
i
on a
ft
ereachpr
egnanc
y
Uni
ver
sal
donor t
op r
eve
ntant
i
-DIgG
of
RBCs;uni
ver
sal f
ormat
ion
Type AB, because they do not make any antibodies. r
eci
pi
entofpl
asma

Hemolyt
icdis
eas
eof Al
sok
nowna
ser
ythr
obl
ast
osi
sfe
tal
is
.
thenewborn
Rhhemol
yti
cdi
seas
eoft
henewbor
n ABOhemol
yti
cdi
sea
seoft
henewbor
n
I
nte
Rac
tIo
n Rh⊝ mot
her
;Rh⊕ f
etus
. Ty
peO mo
the
r;t
ypeAo
rBf
etus
.
me
cha
nIS
m Fir
stpregnancy:mo t
herexpo s
edtofet
al Pre-ex istingma ter na la nt i-
Aa nd/ o ra nt i-BI gG
blood(oft
enduringde l
iv
e r
y)   f
ormat
ionof antibod ie sc ros spl ac ent a   HDNi nt hef etus .
ma te
rnalant
i-DIg G. Usually, Anti-A or Anti-B are IgM, but in blood type O individuals,
Subsequentpre
gna nci
es:anti
-DIgGc ros
sest
he these antibodies can be IgG as well, so they cross the placenta.
placent
a   HDNi nthefetus.
P
ReS
ent
atI
on J
aundic
eshor
tl
yaf
terb
irt
h,k
erni
cte
rus
,hy
drops Mil
djaundic
ei ntheneona
tewit
hin24hoursof
f
eta
lis
. b
irt
h.UnlikeRhHDN,c anoccuri
nfir
st
born
b
abiesandisusual
lyl
esss
ever
e.
t
Rea
tme
nt/
PRe
Ven
tIo
n Prev
entbya dmini
str
ati
onofant
i-
DI gGtoRh Tr
eatnewbor
nwi
thphot
othe
rapyo
rex
cha
nge
⊝p regnantwome nduri
ngthi
rdtrimes
ter t
rans
fusi
on.
andearlypost
par
tum peri
od(i
ffe
t usRh⊕).
Preve
ntsma t
ernalant
i-
DI gGproducti
on.

The prevalence of certain blood types varies widely in different parts of the world.
In the United States, AB-negative is the rarest blood type, and O-positive is the most common.
But this is not universal.
For example, in India, the most common blood type is B-positive, while in Denmark its A-positive.

Since there are 4 ABO classifications and each can be either Rh+ or Rh-, there
are 8 different combinations for blood types.
Prepared for students. Amir Mullick -- YouTube.com/USMLELive Best of luck.
S ectIonI I I He mat ol
ogyandonc ol ogy he matol
ogyandoncol
ogy
—ana
tomy

he
mat
ologyandonc
ology
—ana
tomy

Hemat
opoi
esi
s

Mul
ti
pot
ents
temc
ell

My
eloi
dst
emc
ell L
ymphoi
dst
emc
ell

E
ryt
hropoi
esi
s T
hromb
opoi
esi
s Gr
anul
ocy
top
oie
sis Mono
cyt
opoi
esi
s L
ymp
hop
oie
sis
E
ryt
hro
bla
st Me
gak
ary
obl
as
t My
elo
bla
st Mon
obl
as
t L
ymp
hob
las
t
r
ema
Bon row

R
eti
cul
ocy
te B
and
Me
gak
ary
ocy
te
ood
Bl

E
ryt
hro
cyt
e P
lat
el
et
s E
osi
nop
hil B
asop
hil Ne
utr
oph
il Mon
ocy
te Bc
ell Tc
ell NKc
ell
i
Tsue
s s

Ma
crop
hag
e P
las
mac
ell T
-he
lpe
r T
-cy
tot
oxi
c
ce
ll cel
l

Part of innate immune system.

Neut
rophi
ls Acuteinfamma t
oryres
ponsec e l
ls.Numbe rs Hy pe r
segme ntedneutrophil
s(nucle
usha s6+
A
 i
nba ct
e r
iali
nfecti
ons.Phagoc ytic
. lobe s
)a r
es eeninvit
a minB12/ f
ola
tede fic
ienc
y.
Mul t
ilo
be dnuc l
eusA. Specificgranules Al eftshif
twi t
h b andc ell
s(imma t
ure
contai
nle ukocyt
ea l
kal
inephos phatas
e ne utrophils
)refect
ss t
atesof my el
oid
(
LAP) ,colla
g e
na s
e,ly
sozyme ,a nd pro li
fera
tion(eg,bacter
ialinf
ecti
ons,CML) .
l
actofer
rin.Azurophil
icgranul es(ly
sosomes
) I
mpo r
tantneutrophi
lc hemo ta
cti
ca g
ents: C5a,
contai
npr ot
e i
nases
,acidphos pha t
ase, IL- 8,LTB4, kall
ikr
ein,plat
e l
et-
act
iva
ting
my el
operoxi
da s
e,andβ- gl
ucur onidase
. factor.
A high number of band cells indicates you may be having a bacteremia because your body is making neutrophils to fight some infection.
A patient practicing veganism may have a lack of B12, and that can give hypersegmented neutrophils along with anemia.

Neutrophils-- Mature on left, immature Band cell on right. Hypersegmented neutrophil


in B12 deficiency.

Bone marrow issue. Most likely at or before the common


myeloid progenitor cell because both platelets and RBC
are low and the low reticulocyte count tells that the RBC
are not being replenished.
Prepared for students. Amir Mullick -- YouTube.com/USMLELive Best of luck.
He mat ologyandoncol ogy he
matologya ndonc
ology
—ana
tomy S
ect IonI I
I

Er
ythr
ocyt
es CarryO2t oti
ssuesa ndCO2t olung s.Anucleat
e Er
yth=r
ed;c
yte=c
ell
.
A andlacko r
ganelles;biconcave A, withlar
ge Ery
throc
ytosis=p
olycyt
hemia= Hc
t.
surf
acea r
ea-t
o-volumer a t
ioforrapidg as Ani
socyt
osis=var
yingsi
zes
.
exchange.Lif
es panof1 2 0days.Sour ceof Poi
kil
ocyt
osis=var
yingsha
pes.
energyisgl
ucos e(90%us eding l
yc oly
sis
,10% Anisopoikilocytosis = varying sizes and shapes.
usedinHMPs hunt).Me mb ranesc ontai
n Re
ticul
ocyte=imma tur
eRBC; re
fects
Cl−/HCO3−a nt i
po rt
er,whicha ll
owRBCst o e
rythro
idproli
fer
ati
on.
exportHCO3−a ndt ransportCO2f ro mthe Bl
uishcol
or(poly
chroma s
ia)onWr i
ght
-Gi
ems
a
peri
pherytothel ung sforeli
mi nati
o n. s
tai
nofr e
tic
ulocyt
esreprese
ntsr
esi
dual
r
iboso
ma lRNA.
Young RBC that have just lost their nucleus have large amounts of rRNA, stained via a supervital stain called methylene blue. This helps us detect how young an RBC is. A large amount of
reticulocytes means you are making blood to replace what was lost. If you are not making new reticulocytes, there may be bone marrow malfunction. HbA1c is indicative of the past 120 days.

Thrombocyt
es I
nv olvedi n1 °he mos t
asi
s.Sma llcyt
oplasmic Thromboc yt
openi
ao rp lat
ele
tfunc
tionr
esul
ts
(pl
atel
ets
) fragme ntsA de riv
edf r
om me ga
k a
ryocyte
s. i
np ete
chiae.
A Li f
es pano f8– 10da ys.Whe na ct
ivat
edby vWFr ecept
or:GpIb.
end othe l
ialinjury,aggrega
tewi t
hot her Fi
brinogenrecept
or:GpII
b/II
Ia.
plateletsandi nteractwit
hfib ri
nogentoform Thrombopoieti
nsti
mul a
tesme g
akar
yocyt
e
plateletplug.Co ntai
nde nseg r
anules(ADP , pro
lif
erati
on.
Ca 2+) andαg ranules(vWF ,fibr
inogen, Alf
ag r
anulescont
ainvWF, fi
bri
nogen,
fibr
o nec t
in, pl
a t
eletfa
ctor4).Approximatel
y1⁄
3 fi
bronecti
n,pl
atel
etfa
ctor4.
ofpl atel
etpoo lisstoredinthes ple
en.
A patient on aspirin requires 10 days to recover platelet function so you must wait before performing any surgery. Note that 1/3rd of total platelets are in the spleen, so patients with splenomegaly
may be thrombocytopenic (low circulating platelet count) and patients with no spleen will have an elevated platelet count since that 1/3rd will now be circulating.

Monoc
ytes Fo
undi nb l
ood,dif
fer
enti
at
eint
omacro
phage no=o
s Mo ne(
nuc
leus
);c
yte=c
ell
.
A
i
nt i
ss
ue s
.
Lar
ge,kidney-
shapednucl
eusA.Ex
tensi
ve

frost
edg l
ass
”cy t
opl
asm.

Mac
rophages Phagocyt
osebacter
ia,ce
ll
ulard ebri
s,and Macro=l ar
ge;phage=e at
er.
A se
ne s
centRBCs .Longlif
eint iss
ues. Namedi ff
ersineachti
ssuetype(eg,Kupff
er
Diff
erenti
atefr
omc ir
culat
ingbl ood ce
llsinli
ver,hi
sti
ocyt
esinc onnecti
veti
ss
ue,
monoc yt
esA. Acti
vat
edbyγ -
interf
eron. Langerhanscel
ls[t
ypeofma crophage]i
nskin,
Can f
unc t
ionasantig
en-pr
e senti
ngc ellv
ia os
teoclas
tsinbone,microgl
ialcel
lsinbrai
n).
MHCI I.I
mpor t
antcell
ularco mponento f Li
pidAf rom bact
eri
alLPSb indsCD1 4on
gr
anuloma s(e
g,TB, sa
rcoi
d osis
). macrophagestoini
ti
atesept
icshock.
MHC2 is for adaptive immune response. Macrophages also work
as part of the innate immune response. Macrophages and TNF alpha
are the main actors in granulomatous diseases, involving g-interferon.
Prepared for students. Amir Mullick -- YouTube.com/USMLELive Best of luck.
S ectIonI I I He mat ol
ogyandonc ol ogy he matol
ogyandoncol
ogy
—ana
tomy
Eosinophil cationic protein is aka ribonuclease 3 and is related to inflammation and asthma. Closely related to the eosinophil-derived neurotoxin and both possess neurotoxic, helmintho-toxic, and
ribonucleo-lytic activities. In asthma, eosinophils release histaminase and aryl-sulfatase to limit the allergic reaction after mast cells degranulate.

Eos
inophi
ls Defendagainsthe l
mi nt
hicinfect
ions(major Eo
sin=pi nkdy e;phil
ic=l ovi
ng.
A bas
icp r
otein).Bil
oba t
enucleus.Packed Ca
use sofeosinophil
ia=PACCMAN:
wit
hl ar
gee osi
no phili
cgranulesofunifor
m Parasit
es
s
izeA.Hi ghlypha gocyt
icforantig
en- Asthma
ant
ibodyc ompl exes
. Chur g-
Strausssyndrome
Pr
o d
uc ehista
mi nase,ma j
orbasicprote
in(MBP , Chr onicadrenalinsufficie
ncy
ahe l
minthotoxin),eosi
nophilperoxi
dase, My elopro
lif
e r
ati
vedi s
o rders
eos
inophilca t
ionicprote
in,ande osi
nophil- All
e rgi
cp r
oc es
ses
deri
vedne urotoxi
n. Ne oplas
ia(eg,Hodg kinl ymp homa
)

Bas
ophi
ls Mediat
eall
ergi
creacti
on.De ns
elybas
ophi
li
c Bas
ophil
ic
—s ta
insre
adi l
ywithbasi
cst
ains.
A g
ranulesA cont
ainhepari
n( a
ntic
oagul
ant
) Bas
ophi
li
aisuncommo n,butca
nbeas ignof
a
ndhi s
tamine(v
asodil
ato
r).Leukot
ri
enes myel
opro
lif
era
tiv
edisorder
s,par
ti
cula
rlyCML.
s
ynthes
izedandrele
asedonde mand.

Mas
tcel
ls Me diat
el ocaltis
suea ll
ergi
creacti
ons.Cont ai
n Involvedi
nt ypeIhy perse
nsit
iv
ityreact
ions.
A basophilicg ra
nul e
sA. Ori
ginat
ef r
o ms ame Cr omolynsodium preventsmastcell
precursorasba sophilsbutarenotthesa me degra
nula
tion( usedforast
hmap rophylax
is)
.
celltype.Ca nb indtheFcpor ti
onofI gEt o Va ncomyci
n, opioi
ds,andr a
diocontr
astdyecan
me mbr ane.Ac ti
vatedbytiss
uetrauma ,C3 a eli
ci
tIgE-
ind ependentma s
tcelldegranul
ati
on.
andC5 a,surfaceIgEc ros
s-l
inki
ngbya nti
gen When an allergen is in the body, IgE will bind to the allergen at the binding
(
Ig Ere ceptoragg r
egati
on)   deg
ranula t
ion sites and the mast cell will bind the IgE at the Fc portion. When enough
allergen is present to make IgE cluster and cross link the receptors, the
 rel
easeo fhist
a mine,hepari
n,trypt
as e,
and cells will degranulate, releasing histamine, heparin, and eosinophil
eosinophilc hemot a
cticfac
tors
. chemotactic factors. This is a type 1 hypersensitivity; allergic reaction-
asthma, hay fever, allergic rhinitis.

Dendr
iti
ccel
ls Hi
ghl
yphago
cyt
ica
ntig
e n-
pre
senti
ngc
ell
s(APCs)A.Func
tionasli
nkbet
weeni
nna
tea
nd
A
a
dap
tiv
eimmunesys
tems.Expre
ssMHCc la
ssI
IandFcrecept
orsons
urf
ace
.
Dendritic cells derive from the monocyte. Main inducers of primary humoral immunity; aka antibody response.
Macrophages and B cells are APC too but dendritic cells are best APC.

Top left - Neutrophil


Top right - Lymphocyte
Bottom left - Monocyte

This child has asthma, which is one of the PACCMAN of eosinophils.


Two cells are involved in the pathogenesis of asthma; mast cell and eosinophils.
Eosinophils are mainly known for their MBP.

Heparin is a component of basophils and mast cells as well as an anticoagulant in


type 1 hypersensitivity allergic degranulation. Histamine is also released by basophils
and mast cells and is a vasodilator in allergic degranulation.
Myeloperoxidase is used by neutrophils to kill bacteria. IgE is made by plasma cells.
Prepared for students. Amir Mullick -- YouTube.com/USMLELive Best of luck.
He mat ologyandoncol ogy he
matologya ndonc
ology
—ana
tomy S
ect IonI I
I

L
ymphoc
ytes Re
fertoBc e
lls
,Tc e
lls
,andNKcel
ls
.Bcell
sandTcel
lsmedia
teadapt
iv
ei mmuni
ty.NKcell
sare
A
p
artoftheinnat
eimmuneres
pons
e.Round,de
nse
lys
tai
ningnuc
leuswithsmal
lamountofpal
e
c
ytopla
sm A.
Highest nucleus:cytoplasm ratio, practically 1:1.

The NK cell is the only innate immune system cell from the lymphoid lineage and they act very much like T cells but without the APC and MHC.
It is a special function of these cells to recognize infected cells that specifically do not display MHC markers or have attached antibodies,
and then cause apoptosis of those virally infected or tumor cells.

The B and T cells cannot be differentiated by light microscopy.


They are round cells with densely staining dark purple nuclei and pale, minimal cytoplasm.

Nat
ural
kil
lerc
ell
s I
mpo rt
antininnat
eimmuni t
y,es
pe c
iall
yaga i
nsti
ntra
cel
lularpat
hogens.Largert
hanBa ndT
CD5
6 cel
ls
,withdist
inc
tiv
ecytopl
asmiclyti
cgranules(c
ontai
ningperfor
inandg ranzy
me s
)that
,when
CD1
6(F
cR)
r
eleas
ed,actontar
getcel
lst
oinduc eapoptosi
s.Dis
ti
nguishbetweenhe a
lthyandinf
ecte
dc el
ls
Lyt
i
c
g
ran
ules byide
ntif
yingcel
lsurf
aceprot
eins(i
nducedbys tr
ess
,ma l
ignanttr
ansf
orma ti
on,ormicr
obia
l
i
nfect
ions
).
NKc
ell

Bc
ell
s Mediat
eh umo rali
mmuner esponse.
Origi
nate B=Bo
nema
rrow.
CD2
0 CD2
1 fr
om ste
mc el
lsinbonema r
rowa ndmature
sin
CD1
9 marrow.Mi gr
atetoperi
phera
llymphoi
dt i
ss
ue
(
fol
lic
lesoflymp hnodes
,whitepulpofs
pleen,
Bc
ell
unencapsula
tedlymphoidti
ssue
).Whena nt
igen
i
sencount er
ed,Bc e
llsd
iff
ere
ntiat
eint
op l
asma
The main B cell markers are CD19 and CD20.
cel
ls(
whic hproduceanti
bodi
e s
)andmemo ry
cel
ls.
Ca nfunc t
ionasanAPC.

Tc
ell
s Mediatecell
ularimmuner esponse
.Or igi
nat
e T=Thy mus .
CD8 CD4 fr
oms t
emc ell
sinthebonema r
row, b
utma t
ure CD4 +he lpe rTc e llsa r
et hepr ima ryta rg e
tof
CD3 CD3 i
nt hethymus .Dif
fer
enti
ateintocytot
oxic HI V.
T cel
ls(exp
ressCD8, re
cog niz
eMHCI ), Rul eof8:MHC I I×CD4=8;
T
c T
h
hel
pe rTc e
lls(e
xpres
sCD4 ,re
cognizeMHC MHCI×CD8=8.
II
),andregulato
ryTc el
ls.CD28( cost
imulat
ory
si
gnal)necessa
ryforT-ce
llacti
vat
ion.Mos t T cells function mainly for viral immunity.
Helper T cells have CD4 and interact with MHC2, which are located on the APC.
ci
rculati
nglymp hocy
tesar
eTc el
ls(80%). T cells need the costimulatory signal provided by the CD28 on T cells,
B7 on APC, and CD40L-CD40.

Pl
asmac
ell
s Pro
duc elar
geamo untsofa nti
bodyspeci
ficto Mul
ti
plemy el
omai sap la s
mac e l
ld yscra s
ia.
A
apa r
ti
culara
nt i
gen.“Cloc k-
fa
ce”chroma t
in
Sometimes described as fried-egg appearance.
dis
tri
butio
nande c
ce nt
ricnuc l
eus
,abunda nt Comes from B cells and these make antibodies.
RER, andwell-dev
elopedGol giappa
ratus
(
arrowsinA) .Foundi nbonema r
rowand
norma l
lydonotcirculat
ei nperi
pher
alb lo
od.
Prepared for students. Amir Mullick -- YouTube.com/USMLELive Best of luck.
S ectIonI I I He mat ol
ogyandonc ol ogy he matol
ogya
ndonc ol
ogy
—Phy
S I
ology

he
mat
ologyandonc
ology
—Phy
SIol
ogy

Hemogl
obi
nel
ect
rophor
esi
s

Or
i
gin
Onag e
l, hemoglob i
nmigrat
esfromthe
negati
velyc ha
rg edcat
hodetothepositi
vely
A
A Nor
mal
adu
lt
chargeda node.HbAmi g
rat
esthefart
he st,
A
F Nor
mal
newb
orn fol
lowedb yHbF,HbS,a ndHbC.Thi sis
A
S S
ick
lec
ell
tr
ai
t
becauset hemis s
ensemutat
ionsinHbSa nd
HbCr e placeglutamicac
id⊝ wi t
hv a
line
S
S S
ick
lec
ell
dis
eas
e (ne
utral)a ndl
y si
ne⊕,r e
spect
ivel
y,ma king
A
C HbCt
rai
t HbCa ndHbSmo r
eposi
ti
vel
yc harg
edt han
Hb A.
C
C HbCd
ise
ase
HbS -- Glutamate --> Valine (neutral)
S
C HbS
Cdi
se
ase HbC -- Glutamate --> Lysine (positive)
Glutamate is negative.
C S F A The less negative you become, the further left you remain.
Ca
tho
de A:n
ormal
hemogl
obi
n c hai
n(Hb
A,adu
l)A
t n
ode
F:
norma
lhemogl
obi
nc h a
in(
HbF,f
et
al
)
S:
si
ckl
ecel
lhe
moglo
bin c ha
in(
HbS) AFa
tSa
ntaCl
ausc
an’
tgof
ar.
C:h
emogl
obi
nC c hai
n(HbC)

Hb electrophoresis is used to assess for different forms of Hb in patients with hemoglobinopathy. Normal Hb consists primarily of HbA, which migrates rapidly toward the + electrode (anode) due to
its negative charge. HbS is an abnormal type with glutamate replaced with a neutral valine, decreasing negative charge on the HbS, so it moves slowly to the anode. HbC has lysine, wihch is
positive, so HbC travels even more slowly to the anode. Both HbC and HbS result from missense mutations where a single base substitution results in a codon that codes for different amino acids.
Patients with sickle cell disease have HbS mutations in both beta chains. Those with HbC disease have HbC mutations involving both beta chains. Patients with HbSC disease have 1 HbS and
1 HbC allele, and they will have 2 Hb bands on electrophoresis. Patients who have the trait of a disease, but not the disease will also have 2 Hb bands.
Prepared for students. Amir Mullick -- YouTube.com/USMLELive Best of luck.
Hemat ol ogyandoncol ogy he mat
ologyandonc ol
ogy
—P hy
SI ogy S
ol ect IonI I
I
Some small damage is done to the blood vessel endothelial lining. Platelets adhere with the exposed collagen and vWF binding to the collagen (vWF in the bloodstream also protects inactive
factor ). Platelets bind vWF via the Gp1b receptor. This binding causes platelets to release ADP and Ca from their dense granules. Calcium is key for the secondary hemostasis, and ADP
activates other platelets which adhere to endothelium by inducing the expression of the Gp2b/3a receptor, which binds fibrinogen that becomes fibrin and platelets link and form a clot. TXA2
causes vasoconstriction to lower blood flow and raise platelet aggregation. PGI2 and NO both do the opposite to help balance.
Pl
atel
etpl
ugf
orma
tion(
pri
mar
yhemos
tas
is)
4A 4B

I
NJ URY E
XPOSURE ADHESI
ON ACTI
VATION AGGREGATI
ON
E
n dot
heli
alda
mage v
WFb i
ndst
oexposed Pl
ate
let
sbi
ndvWFvi
aGp I
b ADPbin
d i
ngtoP2Y1
2 Fi
br
inoge
nbind
sGp
II
b/
II
I
are
cep
tor
san
dli
nksp
lat
el
et
s
→t ra
nsi
ent c
oll
agen re
cept
oratt
hesi
teofi
nj
ur
y re
cept
or i
ndu
c e
sGp I
Ib
/II
I
a Ba
lan
ceb
etween
v
a s
o c
onst
ri
cti
onvi
a v
WFisfr
omWe i
bel
-Pal
ade onl
y(sp
eci
fic)→p l
at
el
ets expr
ess
ionatpl
ate
let
n
e ura
lst
imula
ti
onrefl
ex under
goconf
ormat
i
onal sur
fa
ce Pro-
aggr
egat
ionf
act
ors
: An t
i
- a
ggreg
ati
onfa
cto
rs:
b
odie
sofendot
hel
ial
a
n dendoth
eli
n(r
el
eased c
ell
sandα-g
ranul
esof chan
ge TXA2(
rel
eas
ed PGI
2andNO( r
el
eas
ed
f
romd amagedcel
l
) p
lat
el
ets bypla
tel
et
s) byendot
hel
ia
lcel
l
s)
↓b l
oodflow ↑bloodflow
Pla
tel
et
srel
eas
eA DPand ↑p l
at
ele
taggr
ega
ti
on ↓plat
ele
taggr
ega
ti
on
Ca2+(
nece
ssa
ryfor
coagu
lat
i
oncascade)
,TX
A2
T
emp
ora
ryp
lugs
top
sbl
eed
ing
;un
sta
ble
,ea
si
lyd
isl
odg
ed

A
DPhel
psp
lat
el
et
sad
her
e
t
oen
dot
hel
ium Coa
gul
at
ionc
asc
ade
(
sec
onda
ryhe
mosta
sis
)

Thr
ombogenes
is Forma ti
onofi nsolublefibrinme sh.
Aspir
ini rreversi
blyi nhibit
sc ycloo xygenase,
Bernard Soulier -Lack of Gp1b - therebyi nhibiti
ngTXA2s ynthe sis.
Prevents platelet adhesion to vWF and stops adhesion, activation, and aggregation of platelets.
vWF Disease - Lack of vWF - Clopidogre l,
pr asugrel,andt icl
op idineinhi bi
t
Same effects as Bernard Soulier.
Ristocetin fails in both of the above diseases; negative agglutination test. ADP- i
nd uce dex press
ionofGpI Ib/III
aby
irr
eve r
si
b lyb lo
c kingP2 Y12r ece ptor.
Clopidogrel, prasugrel, ticlopidine block the ADP receptor, preventing ADP from platelets from binding,
thus blocking Gp2b/3a release. Abciximab, eptifibatide, tirofiban inhibit Gp2b/3a. Abcixima b, epti
fibatide
,a ndt i
ro fibaninhi bit
Glanzmann Thrombasthenia is a lack of Gp2b/3a.
GpI Ib/I
II ad i
rectl
y .
Rist
oc et
ina c
t i
vatesv WFt ob indGpI b.Fa i
lure
Cl
opi
dogr
el
,pra
sug
rel
,
ofag gre
g ati
onwi thr i
stocetina ssayoc cursin
P
lat
el
et
t
ic
lopi
di
ne vonWi ll
e branddi seasea ndBe rna r
d-So ul
ier
In
sid
e v
WF syndrome .
A
spi
r
in p
lat
el
ets

bri
nog
en vWFc arries/
pr ot
e ctsfa
c t
orVI I
I ;v ol
ksWa ge n
F
ibr
i
nog
en Factoriesma keg r8c a
rs.
C
OX
A
rac
hid
onic T
XA2
A
DP(
P2Y
12)r
ece
pto
r
a
cid
4B
4A
De
fici
enc
y:Gl
anz
man
nth
romb
ast
hen
ia
Gp
II
b/
II
I
a

A
cti
va
ted
De
fici
enc
y:Ber
nar
d- Abci
xi
mab, T Pro
tei
nC p
rot
ei
nC
GpI
Ib
/I
II
a hro
mb i
n-
S
oul
i
ersyn
drome ept
ifi
bat
ide
, t
hr
ombomodu
li
n V
asc
ula
ren
dot
hel
i
alc
ell
s
in
ser
ti
on
ti
r
ofiban comple
x
Defi
cie
ncy:v
on
Wi
ll
ebra
nd I
nsi
de (
vWF+fact
orV
II
I)
S
ubend
othel
i
al Gp
Ib v
WF di
se
ase e
ndot
hel
ia
l t
hr
ombopla
sti
n
c
oll
age
n c
ell
s t
PA,
PGI2

UWORLD= A 17yo girl comes to the office for follow up on anemia. She has taken iron supplements regularly since being diagnosed with iron deficiency anemia 3 months ago. However, she
still feels fatigued and does not think the supplements have improved her symptoms. She has occasional gingival bleeding when brushing her teeth. Menses occur every 27-28 days and last 7-8
days with heavy flow, sometimes requiring her to change pads every hour. Platelet count is normal. Further evaluation reveals that the patient's platelets do not agglutinate appropriately in
response to ristocetin. When normal plasma is added to the solution of her platelets and ristocetin, appropriate agglutination occurs. What is the most likely deficient in her? Von Willebrand Factor.
vWF is an important hemostatic glycoprotein synthesized by endothelial cells and megakaryoctyes. Following endothelial damage, vWF bdings GP1B receptors on platelets and mediates
platelet adhesion to subendothelial collagen. Deficiency of vWF can lead to easy bruising and prolonged mucocutaneous bleeding. Lab workup reveals a normal platelet count and an abnormal
ristocetin assay. The ristocetin assay measures in-vitro vWF dependent platelet agglutination by activating GP1B receptors on platelets. Combined oral contraceptives are used for treatment of
menorrhagia due to vWF deficiency, and desmopressin can be used to stimulate vWF release from the endothelium.
Prepared for students. Amir Mullick -- YouTube.com/USMLELive Best of luck.
S ectIonI I I He mat ol
ogyandonc ol ogy he matol
ogya
ndonc ol
ogy
—Phy
S I
ology

This breakdown pathway starts with prekallikrein becoming kallikrein thanks to factor XIIa. Kallikrein will cause clot
breakdown and enhance inflammation via conversion of HMWK to bradykinin, which causes the effects below. Kallikrein
also converts plasminogen to plasmin, which converts c3 to c3a which begins the complement cascade to fight infections
Coagul
ati
onandki
ninpat
hways and reduce inflammation. Plasmin also breaks the fibrin mesh into degradation products like D-dimers.

Col
lage
n , HMWK
bas
emen tmembr
ane
,
K
all
i
kre
in ↑V
aso
dil
at
io
n
act
iv
atedpla
tel
et
s

Cont
act B
rad
yki
ni
n ↑P
erme
abi
l
it
y
act
iv
ati
on
(
int
ri
nsi
c) X
II X
II
a Ki
ni
nca
sca
de ↑P
ain
pat
hway X
I X
Ia Rubor, Calor, Dolor, and Tumor.
Heparin has greatest anticoagulation efficacy on
thrombin while LMWH has greatest efficacy on factor Xa. I
X I
Xa
T
is
suef
act
or * VII
I
*VI
I
Ia wi
t
hv WF ANTI
COAGULANTS:I
Ia(
thr
ombin
)
T
iss
uefa
ctor VI
I V
II
a -hepa
ri
n
(
ext
ri
nsi
c) * * -LMWH
p
athway X * Xa -di
rec
tthr
ombini
nhi
b i
t
ors(
eg,
arga
tr
oba
n,
*Va V bi
val
ir
udi
n,dabi
gat
ran
)
ANTICOAGULANTS:f
act
orXa
-LMWH( eg
,dal
tepar
i
n,eno
xapa
ri
n) II * IIa
-hepar
in P
rot
hro
mbi
nTh
rombi
n P
las
min
oge
n
-di
rectXai
nhi
bit
ors(
eg,
api
xaba
n) T
HROMBOLYTICS:
-f
o n
dapari
nux a
lt
epl
as
e,r
etep
lase
,
I I
a t
PA s
tr
ept
oki
nas
e,t
e n
ecte
pla
se
F
ibr
i
noge
nFi
br
inmono
mer
s
A
NT I
FI
BRINOLYTI
CS:
A
ggr
ega
ti
on
P
las
min a
minocap
roi
cac i
d,
Most common --> He
mop
hil
i
aA:
defi
cie
ncyo
ffa
cto
rVI
I
I(X
R) t
ra
nexami
ca c
id
He
mop
hil
i
aB:
defi
cie
ncyo
ffa
cto
rIX(
XR) Combi
ned
He
mop
hil
i
aC:
defi
cie
ncyo
ffa
cto
rXI(
AR) pat
hway Ca
2+
X
II
Ia X
II
I F
ibr
inol
yt
ics
yst
em
No
te:
Kal
l
ik
rei
nac
ti
vat
esb
rad
yki
ni
n;A
CEi
nac
ti
vat
esb
rad
yki
ni
n
*=r
equ
ir
eCa
2+
,p
hos
pho
li
pi
d F
ibr
i
nd e
gra
dati
on
=i
nhi
bi
tedbyvi
ta
minKant
agon
is
twar
fa
ri
n p
roduc
ts(
eg,D-
dime
r)
=co
fac
tor F
ibr
i
nme shs
tab
il
iz
es
=ac
ti
vat
esbut
notpar
tof
coagu
lat
i
oncas
cad
e p
lat
el
etpl
ug
L
MWH=l
ow-mol
ecul
ar-
wei
ghthe
par
in
All steps in the radius of 10-->10a and the
fibrin mesh network need Calcium; 2, 7, 9, 10.

Extrinsic path -- Starts at factor 7 which is activated by thromboplastin (tissue factor). Then 7-->7a and 7a activates
factor 10-->10a which is part of common path. This is an important path for clot initiation. We measure its function by
following Prothrombin Time (PT) by adding tissue factor to activate factor 7 and see if it works. The other test is the INR.

Intrinsic path -- Starts at factor 12, which is activated when its exposed to collagen, basement membrane, platelets, or
HMWK. Then we get 11-->11a, 9-->9a, and the 9a works with the cofactor 8a. The factors 9a and 8a lead to our common
pathway factor 10. This is best monitored by the Partial Thromboplastin Time (PTT) initiated by phospholipids and an
activator substance that tricks factor 12 to be activated.

Common path -- Starts with 10-->10a, and 10a requires the cofactor 5a to function. They activate factor 2-->2a
(prothrombin-->thrombin) and thrombin breaks fibrinogen to fibrin monomers, 1-->1a.
Thrombin also activates 5, 8, 9, and 13. Factor 13-->13a further stabilizes the fibrin mesh network.
Proteins C and S stop the cofactors 5a and 8a.

Patient is on long term broad spectrium antibiotic therapy. That can


kill normal gut flora needed for vitamin K synthesis, which is needed for
2 7 9 10 c & s. Factor 7 is measured via PT for extrinsic so only PT would be
prolonged if 7 is deficient. 8, 11, and 12 are part of intrinsic and would only
prolong PTT, but remember 8 is safe in vWF. 5 doesnt require K.

2 requires K and is part of common path so it can prolong both PT and PTT.
Prepared for students. Amir Mullick -- YouTube.com/USMLELive Best of luck.
Hemat ol ogyandoncol ogy he mat
ologyandonc ol
ogy
—P hy
SI ogy S
ol ect IonI I
I

Vi
tami
nK–dependentc
oagul
ati
onc
omponent
s
Pr
ocoagul
ati
on Vi
taminKde fcie ncy:s ynthe si
soffactorsII,
P
roc
oag
ula
ti
on VII,IX,X, proteinC, pr oteinS.
Re
duc
ed Wa r
farini nhi bitsvit
a minKe poxidereduc t
ase.
v
it
ami
nK I
nac
ti
veI
I
,VI
I
,IX
,C,
S
(
act
i
ve) Vitami nKa dmi nistr
atio nca npotenti
a ll
y
-
glu
tamyl
car
boxy
las
e
(
vi
tami
nK-de
pen
dent) r
e ver
sei nhi b i
torye ff
ectofwa rfari
no nc lot
ti
ng
War
fa
ri
n, E
poxi
de f
a c
tors ynt he s
is(delayed) .FFPo rPCC
Mat
ur
e,ca
rbo
xyl
at
ed
l
i
ver
fai
l
ure r
edu
cta
se I
I
,VI
I,
IX,
X, C,S admini strationr everse
sa ct
iono fwa rf
arin
Oxi
di
zed i
mme diatelya ndc anbeg i
ve nwi t
hv i
tami nK
v
it
aminK
(
i
nac
ti
ve) Cl
ott
i
ng A nti
- i
nc ase so fse v
e rebleedi ng.
L
ive
r
fa
ctor
s c
oag
ulant
s Neonate slac ke ntericba cteri
a, whichpr oduce
vit
ami nK.Ea r
lya dmi nistrat
ionofv it
a minK
overcome sne ona taldeficiency/coagulopathy.
Protein C has a shorter half life.
STEP1 is too long (2 - longest).
Fact
orVI I(Se ve n)—Shor tes
tha lfli
fe.
F
ibr
i
nog
en F
ibr
i
n
Snow White had 7 dwarfs (7 - shortest). Fact
orI I(Two )—Long e
s t(Tallest
)ha l
flife.
Ant
icoa
gul
ati
on Ant i
thrombininhibi
tsthrombi n(fac
torIIa)a
nd
f
actorsVIIa
, I
Xa,Xa ,XIa,XIIa.
He parinenhancesthea cti
vi
tyo fant
ithrombin.
An
ti
coa
gul
at
io
n
Principalt
arget
sofa nti
thrombin:thrombina nd
Th
romb
in
f
actorXa.
in FactorVLe idenmut ati
o nproducesaf ac
torV
He
par
i
n-l
i
kemo
lec
ule b Pro
(
enh
anc
esA
TII
Iac
ti
vi
ty
) rom ay pa t ei
n
r
esist
anttoinhib
iti
onb ya cti
vate
dp rote
inC.
th th
n
i
t thw wa C T
hrombin- tPAisus edcli
nical
lyasat hrombolyti
c .
A pa y
A
nti
t
hromb
inI
I
I t
hr
omb omodul
i
n
c
omp l
ex
(
endot
hel
ial
cel
l
s)
I
nhi
bi
tst
hrombi
n
(
andVI
Ia
,IXa
,Xa,X
Ia,
XII
a) P
rot
ei
nC A
cti
vat
edp
rot
ei
nC Factor V Leiden. 5% of white population is heterozygous for this mutation
and have a 10% lifetime risk for DVT, which is huge. Feels like excess
R
equi
re
s factor V which causes excessive clotting so you should suspect DVT
p
rot
ei
nS or pulmonary embolism without any risk factors like smoking or OCP.

Cl
eav
esan
d
i
nact
i
vat
esV
a,VI
I
Ia
Prepared for students. Amir Mullick -- YouTube.com/USMLELive Best of luck.
S ectIonI I I He mat ol
ogyandonc ol ogy he matol
ogya
ndonc ol
ogy
—Pa
thology

he
mat
ologyandonc
ology
—Pa
thol
ogy

Pa
thol
ogi
cRBCf
orms
t
yPe e
XamP
le a
SSo
cIa
tedP
ath
olo
gy n
ote
S
Acanthoc
ytes A Li
verdi
seas
e, Ac
a ho=s
nt piny
.
(
“spurcel
ls
”)A a
betal
ipo
prot
ei
nemi
a(sta
tesof
c
holes
ter
oldy
sre
gul
ati
on)
.

Dacrocyt
es B Bonema rr
owinfil
tr
ati
on(e
g, RBC “s
hedsate
a r
”becausei
t’
s
(
“tear
dropcel
ls
”)B myel
ofibr
osi
s)
,thala
sse
mias. mechani
cal
lys
que e
zedoutofi
ts
ho
mei nthebonema r
row.
Seen in beta thalassemia major or myelophthisic anemia due to myelofibrosis, which is a rare bone marrow cancer in
which an abnormal clone of hematopoietic stem cells in the bone marrow and other sites results in fibrosis of marrow.
Could be due to mutation in JAK2, Janus Kinases, non-receptor tyrosine kinases essential for activation of signaling
that is mediated by cytokine receptors lacking catalytic activity.

Degmacyt
es C G6
PDde
fic
ienc
y. Duet
oremovalofHei
nzbodi
esby
(
“bi
tec
ell
s”)C mac
ropha
gesinthes
ple
en.

Ec
hinocyt
es D End-stager
enaldis
ease
,li
ver Diff
ere
ntfromaca
nthoc
yte;it
s
(
“bur
rcell
s”)D di
se as
e,pyruv
atekina
se proj
ect
ionsar
emo r
eunif
o r
ma nd
defic
iency
. small
er.

El
li
ptoc
ytesE E Her
edit
a r
yell
ipt
ocyt
osis
, usua
lly
as
ymptomati
c;causedb y
muta
tioningenesencodingRBC
membr a
neprote
ins(eg,spec
tri
n).

Mac
ro-
oval
ocyt
esF F Megal
obl
ast
ica
nemi
a(a
lso
hy
pers
egment
edPMNs)
.
Prepared for students. Amir Mullick -- YouTube.com/USMLELive Best of luck.
Hemat ol ogyandoncol ogy hematologyandoncol
ogy—P a
thol
ogy S
ect IonI I
I

Pat
hol
ogi
cRBCf
orms(
cont
inue
d)
t
yPe e
XamP
le a
SSo
cIa
tedP
atho
log
y n
ote
S
Ri
ngeds
ider
obl
ast
sG G Si
der
oblas
ti
ca ne
mia.Exces
sir
on See
ni nsi
deb onema rr
ows mear
i
nmi t
ochondri
a(per
inuc
lear wit
hs peci
alsta
ining(Pruss
ian
r
ing
). bl
ue),vsbaophi
s l
icst
ippli
ngin
per
ipherals
me ar.

Sc
his
toc
ytesH H Microang i
opathichemolyt
ic Fr
agment
edRBCs(
eg,he
lme
t
anemia s
,includingDIC,TTP/ c
ell
s)
.
HUS, HELLPs yndr
ome ,
mec hanicalhemo l
ysi
s(e
g,hea
rt
val
vepr ost
he s
is
).

Si
ckl
ecel
l
sI I Si
ckl
ece
lla
nemi
a. Si
ckli
ngo c
curswi
thd
e hy
drat
ion,
deoxyg
enati
on,a
ndathig
h
al
tit
ude.

Spher
ocyt
esJ J Her
edit
arys
pher
ocy
tos
is
,drug-a
nd Small
,spher
icalc
ell
swi
tho
ut
i
nfe
cti
on-i
nduc
edhemoly
tic ce
ntralpa
llor
.
a
nemia.

T
argetc
ell
sK K HbCdiseas
e,Asp
leni
a,Li
ver HALT,
“ ”sa
idt
hehunt
ert
ohi
s
di
sea
se,Thal
ass
emia. t
arg
et.

DIC
Prepared for students. Amir Mullick -- YouTube.com/USMLELive Best of luck.
S ectIonI I I He mat ol
ogyandonc ol ogy he matol
ogya
ndonc ol
ogy
—Pa
thology

RBCi
ncl
usi
ons
t
yPe e
XamP
le a
SSo
cIa
tedP
ath
olo
gy n
ote
S
Bas
ophi
li
cst
ippl
ingA A Si
derobl
ast
icanemia
s(eg,l
ead Seenprima r
ilyi
npe r
ipher
alsmear
,
poi
soni
ng, myel
odys
pla
sti
c vsri
ng e
ds ider
obl
ast
sseenin
sy
ndromes),t
hal
ass
emias. bonema rrow.
Aggre
gati
ono fri
bos
oma lp
reci
pi
ta
tes
.
Dono tcontai
niro
n(incont
ras
tto
Pappenhei
me rbodi
es)
.
Hei
nzbodi
esB B Se
eni
nG6PDde
fic
ienc
y. Oxida
tives
tre
ss  Hbde nat
ures
andpreci
pit
ates(
He i
nzbodie
s).
Phagocyt
icremovalofHei
nz
bodi
e s  bi
tecel
ls
.

Howel
l-
Jol
l
ybodi
esC C See
ni npati
ent
swi
thfunct
iona
l Bas
ophil
icnuc
lea
rremnant
sfound
hypospl
eni
aoras
ple
nia. i
nRBCs .
Howel
l-J
oll
ybodi
esar
eremoved
f
rom RBCsbyspl
enic
macr
ophages
.

Pappenhei
mer D Si
deroc
ytescont
aini
ngbas
ophi
li
c Dist
inctfr
o m baso
phili
cst
ipp
ling
bodi
esD gra
nulesofi
roninsi
der
obl
ast
ic (f
orme dfrom ri
bosomal
anemias
. preci
pita
tes/noir
on)andHeinz
bodies(contai
niron/
dena
tured
hemo glo
bi n
).
Prepared for students. Amir Mullick -- YouTube.com/USMLELive Best of luck.
Hemat ol ogyandoncol ogy hematologyandoncol
ogy—P a
thol
ogy S
ect IonI I
I

Anemi
as
An
emi
as

Mic
roc
yti
c Nor
mocyti
c Ma
croc
yti
c
(
MCV<80f
L) (
MCV80–1
00fL) (
MCV>100fL
)

He
mog
lob
ina
ffe
cte
d(T
AIL
) Me
gal
obl
ast
ic Non
meg
alob
las
ti
c
Def
e ct
iveglobi
nchain:
•Th
alassemias DNAa
ffe
cte
d
Def
e ct
ivehemes y
nthesi
s: Defec
ti
veDNAsynthe
sis •
Dia
mond-Bl
ack
fana
nemi
a
•Anemiaofchroni
cdi
s e
ase •Fol
at
edefici
enc
y •
Li
verd
ise
ase
•I
rondefic i
ency •Vi
ta
minB 2de
ficie
ncy
1 •
Alc
ohol
is
m
•Le
adp ois
on i
ng •Orot
i
caci
duri
a
Defec
ti
veDNArepai
r
•Fa
nconi
anemia

Nonhemol
yt
ic Hemoly
ti
c
(
re
ti
culocy
tei
nde
x≤2%) (
r
eti
cul
ocyt
ein
dex>2
%)


Ir
ondefici
enc
y(ear
ly
)

Anemiao
fchr
o n
icdi
sea
se

Apl
ast
ica
nemia I
ntr
ins
ic E
xtr
ins
ic

Chron
icki
dneydi
se
ase
Memb r
aned ef
ects •
Autoi
mmu n
e
•Her
edi
taryspher
ocyt
osi
s •
Micr
oangi
opat
hi
c
•Par
oxys
ma lnoct
urna
l •
Macroa
ngio
pat
hic
he
mog l
ob i
nuri
a •
Inf
ect
ion
s
Enzy
med efic i
enci
es
•G6PDdefic i
ency
•Py
ruvat
ek i
nasedefi
cie
ncy
Hemoglobinopat
hie
s
•Si
ckl
ecel
l ane
mia
•HbCdis
ease
Prepared for students. Amir Mullick -- YouTube.com/USMLELive Best of luck.
S ectIonI I I He mat ol
ogyandonc ol ogy he matol
ogya
ndonc ol
ogy
—Pa
thology
In microcytic anemia, the RBC are smaller and have a lesser red color. They also have more central pallor. Can be due to problems with making heme or problems with making globin.
Globin issues are thalassemias, while heme issues are usually iron deficiency. Lead poisoning and sideroblastic anemia are also causes.

Micr
ocyti
c,
hypochr
omicanemi
as MCV<80f
L.
I
rondefic
iency  
ironduet ochr onicbl eeding(eg,GIlo
s s
,me no r rha g ia )
,ma lnut ri
tio n,a bs or p t
iondi sord ers,GI
Imagine a patient under 50 age with melena surger
y( eg,gastrec
to my),or  demand(eg ,pre gna nc y )    fina lst
e pi nhe mes ynt hesis.
and IDA. Remember melena is black tarry
stool due to oxidized blood in feces, which Labs: iron,  TI BC,  fe
r r
iti
n,  f
reeer
y t
hro cy tepr ot o po rphy rin,  RDW.Mi c roc ytos
isa nd
indicates bleeding in upper GI, and the most hypochroma si
a(  centralpallor
)A.
common cause is peptic ulcer disease. If the
patient is over 60 with hematochezia, you Symptoms :fat
igue ,conjunc ti
valpal
lorB, pi
c a( pe rs ist e ntc rav inga ndc ompul sivee ati
ngo f
need a colonoscopy because colon cancer is nonfoods ubst
a nces),spoo nnail
s(koi
lonychia).
likely. In pregnancy, blood volume increases
by 50% and their RBC mass by 33%, Ma yma nif
e s
tasg lossi
tis,cheil
osi
s,Pl
ummer -Vins ons y ndr ome( tri
a dofi ronde fic
i encya nemi a
,
After initial treatment for IDA, patients will have immature RBC
drastically increasing iron demand. esophagealwe bs,anddy sphagia
). with Wright-Giemsa blue rRNA in their reticulocytes.
α-
thal
ass
emi
a α-g
lob
ingenede
let
ions   α-globi
nsynt
hesis
.csd
i elet
ion(
del
eti
onsoc
curonsamechr
omos
ome
)
TIBC - Transferrin Iron Binding Capacity pr
eval
enti
nAsi
anpopulat
ions;tansde
r l
eti
on(del
eti
onsocc
uronsepa
rat
echromos
omes)
Transferrin binds iron tightly but reversibly.
When Iron is low, you make more pr
eval
enti
nAfri
canpop
ulatio
ns .
Norma sαα/
li αα.
transferrin so the transferrin saturation
decreases because there is less iron, n
UmB
e Fα-
Ro glo
BIng
ene
Sde
let
ed d
ISe
aSe c
lIn
Ica
loU
tco
me
but TIBC rises. When iron is high,
transferrin decreases.
αα/
1( α–) α-
tha
las
semi
ami
nima Noa
nemi
a(s
il
entc
arr
ier
)
Ferritin is an indirect marker of total α–
2( α–
/ ;t
ra )or
ns α-
tha
las
semi
ami
nor Mildmicrocyt
ic,hypo
c hr
omi
c
iron in body. It stores iron to release αα/
( ––;c
i)
s anemia;csd
i eleti
onma y
in a controlled fashion and is
considered an iron guard. wors
enoutcomef orthe
car
rie
r’
soffs
pring
3(
–––α)
/ Hemogl
obi
nHdi s
eas
e(HbH)
; Moder
atet
osev
eremi
croc
yti
c
e
xce
ssβ-g
lobi
nf msβ4
or hy
pochro
micanemi
a
4(
––/
––) Hemogl
obinBar
tsdis
eas
e ; Hydr
opsf
eta
li
s;i
nco
mpa
tib
le
noα-g
lobi
n,e
xcessγ-
globi
n wit
hli
fe
f
ormsγ4
β-
thal
ass
emi
a Poi
ntmut ationsins plicesi
tesandpr omo ters equences    β-gl
obins y
nthe si
s.Preva
lentin
The Kozak consensus sequence is Me dit
e r
ra neanpop ulati
ons.
analogous to the Shine-Delgarno
sequence in E coli. It is defined by the β-
thalassemi ami nor( he t
erozygote
):βc haini sund erpr
od uced.Us uall
yasy mp t
oma t
ic.Diagnosi
s
following mRNA sequence:
(gcc)gccRccAUGG, in which R is confir
me dby   HbA2( >3 .5
%)one lect
rop ho res
is.
either A or G. β-
thalassemi ama jor( homo z
ygo t
e):βcha i
ni sabsent   s
everemi crocyt
ic,hypoc hr
omi c
Among other factors, a purine (A or G) anemi awi thtargetc ell
sa ndincreasedani sopoikilo
cytosi
sC r equiri
ngb loodt r
ansfusi
on(2°
positioned 3 bases upstream from AUG
appears to play a key role in this initiation hemo chr oma t
osis).Ma rrowe xpansi
on( “crewc ut”onskullx-ray)  skel
e t
alde f
ormi t
ies(
eg,
process. If the R is a C or T, the initiation “chi
pmunk”f aci
e s).Extramed ull
aryhema topo i
esis  hepa t
ospl
enome ga l
y.  ri
skofpa rv
ovir
us
of translation will be hindered, leading to
defective proteins such as B19–ind uc edaplasticc r
is
is. HbF( α2γ2)
, Hb A (α
2 22δ).Hb F i
s pr
o te
cti
ve in t
he inf
a ntanddis
ease
the case in beta thalassemias.
become ss ympto ma ticonlyafter6mont hs ,whe nfet
alhe mo gl
obinde cl
ine s
.
HbS/β-thala s
semi ahe terozygot :
emi l
dt omo derat
esic
k l
ec el
ldiseasedepe ndingona mountof
β-gl
ob inp roduction.
There are 4 copies of the alpha gene, 2 from mom and 2 from dad. Configuration is key. Cis configuration; 2 mutations from same
parents' chromosome, so 2 patients with mutations in cis formation may have a child with 0 copies of alpha gene because he would
inherit 2 complete mutations from both parents, giving 4 mutations. Fetus dies and you see hemoglobin Bart, with 4 gamma.
Trans; 2 mutations but each on opposite chromosomes. Thus, potential fetus would get 1 fine and 1 mutated gene from both parents,
so child would have 2 mutated alpha no matter what. To have 3 mutations, 1 parent must give 2 mutations in cis
and 1 from a trans; Hemoglobin H. There will be some normal alpha and the beta will form beta globin tetramers, which is HbH.

Beta Thalassemia. There are 2 copies of the gene, 1 from mom, 1 from dad. Severity of disease is proportional to the number of mutations. In beta minor you have 1 mutation and patients are
relatively asymptomatic. You will note more HbA2 = 2alpha, 2delta. In beta major, there is severe anemia. Since there is high demand for new RBC, patients use bone marrow outside the axial
skeleton (skull) leading to crew cut x-rays and chipmunk facies.
Prepared for students. Amir Mullick -- YouTube.com/USMLELive Best of luck.
Hemat ol ogyandoncol ogy hematologyandoncol
ogy—P a
thol
ogy S
ect IonI I
I

Mi
croc
yti
c,hypoc
hromi
canemi
as(
cont
inued)
L
eadpoi
soni
ng Leadinhibi
tsfe rr
oche l
atasea ndALAde hydrata
se    hemes y
nthesi
sand  RBCpr otopo
rphy
rin.
Alsoinhib
itsr RNAde gradation   RBCsr e
ta i
na g
gregat
esofrRNA( bas
ophil
icst
ippli
ng).
Although lead poisoning usually occurs in kids, it
can also occur in adults. Affected individuals are Symptomso fLEAD p ois
oni ng :
usually miners or industrial workers at battery LeadLine so ngingivae(Bur tonlines
)andonme taphys
esoflongbonesD onx-r
a y
.
manufacturing factories. Adults present with
weakness, abdominal pain, constipation, and Encephalopa t
hya ndEr yt
hr ocyt
eba s
ophil
icstippl
ing.
neurologic manifestations. Basophilic stippling
(RBC with little blue dots all over) are key.
Abdomina lcolica ndsider
ob last
icAnemi a
.
Drops—wr istandf ootdrop .Dime rc
aprolandEDTAa re1stl
ineoftre
atment.
Succimeruse dforc helat
ionf orkids(I
t“sucks”tobeak i
dwhoe a t
sle
ad).
Exposureri
sk  inoldhous eswi thc hi
ppedpa i
nt.
Si
der
obl
ast
ica
nemi
a Causes
:genet
ic(eg,X-l
inkeddefe
ctinALAs ynt
ha s
egene),ac
quired( myel
odyspl
ast
icsyndromes),
a
ndr ever
si
ble(al
coholismo s
tcommo n;al
solead,vi
ta
mi nB6de fici
ency
,copperdefic
ienc
y ,
drugs
[
eg,is
oniazi
d,li
nezoli
d]).
La
bfindings: ir
on,norma l
/ TIBC,  fe
rri
ti
n.Ringedsid
e r
obla
sts(withir
on-la
den,Prussi
an
bl
ue–sta
inedmi t
ochondria
)seeninbonema rr
ow E.Pe r
iphera
lbl oodsmear:bas
ophili
cstippl
ing
o
fRBCs .
Tr
eatment:pyr
idoxi
ne(B6, cof
act
orforALAsynthase
).
A B C D E

UWORLD = A 40yo woman comes to the office with a 3 month history of progressive limitation of physical activity due to fatigue. She says, "I could barely walk from my car to your office." Past
medical history is significant for a positive tuberculin skin test 7 months ago with a normal chest radiograph. She has been compliant with the prescribed treatment despite its bitter taste.
Physical exam shows a tired-appearing woman with conjunctival pallor. Results of a CBC show Hb=9, Hematocrit=28%, and MCV=72. Bone marrow aspirate revealed the image given below
under Prussian blue stain. Decreased activity of which enzyme most likely explains her anemia? D-ALA Synthase. She has latent TB with lab values and a bone marrow aspirate consistent with
sideroblastic anemia due to isoniazid use. Sideroblastic anemia is diagnosed by bone-marrow exam with Prussian blue stain. Causes include X-linked sideroblastic anemia due to
D-ALA Synthase mutation, myelodysplastic syndrome, alcohol abuse, copper deficiency, and certain medications (isoniazid, chloramphenicol, linezolid).

Isoniazid directly inhibits the enzyme pyridoxine phosphokinase, which normally converts pyridoxine (B6) to its active form, pyridoxal 5' phosphate, a cofactor of D-ALA Synthase. Inhibition of the
enzyme produces a microcytic, hypochromic anemia. Iron is transported to developing erythrocytes that cannot form heme, and its granules accumulate circumferentially around the nucleus,
forming ring sideroblasts. The pyridoxind deficiency can also lead to dermatitis, stomatitis, neuropathy, and confusion, so give B6 with isoniazid.

Hepcidin is an acute phase reactant synthesized by the liver that acts as the central regulator of
iron homeostasis. High iron levels and inflammatory conditions increase the synthesis of hepcidin,
while hypoxia and increased erythropoiesis act to lower hepcidin levels. Hepcidin influences body
iron storage through its interaction with ferroportin, a transmembrane protein responsible for
transferring intracellular iron to the circulation. Upon binding hepcidin, ferroportin is internalized
and degraded, decreasing intestinal iron absorption and inhibiting the release of iron by
macrophages.

Iron absorption from the proximal small intestine is facilitated by the divalent metal transporter 1,
DMT1. Once inside the intestinal cells, iron may bind ferritin and remain stored within the
enterocyte or iron may enter circulation through ferroportin and free iron released into circulation
is transported throughout the body by transferrin.
Prepared for students. Amir Mullick -- YouTube.com/USMLELive Best of luck.
S ectIonI I I He mat ol
ogyandonc ol ogy he matol
ogya
ndonc ol
ogy
—Pa
thology

Mac
roc
yti
canemi
as MCV>1
00f
L.
d
eSc
RIP
tIo
n F
IndI
ngS
Megal
obl
ast
icane
mia I
mpa i
redDNAs ynt
hes
is ma t
urat
ionof RBCmacr
ocy
tos
is
,hyper
seg
ment
edne
utr
ophi
ls
A nucl
eusofpr
ecurs
orce
llsi
n bonema r
row (
arr
owi
nA),gl
oss
it
is
.
de
layedr
ela
tiv
etomaturat
ionofcyt
oplas
m.

DNA synthesis is not keeping up with cell growth, so cells grow more with each division.
When these cells make it to circulation, they will be fewer in number, larger in size, and less mature.

F
ola
tede
fci
enc
y Cause
s:ma lnutr
iti
on(eg ,a
lcohol
ics
), homocy
ste
ine
,norma
lmet
hyl
ma l
onica ci
d.
malabsor
ption,drugs(eg,methot
rexa
te, Noneurol
ogi
csympt
oms(
vsB12defic
iency)
.
tr
imethoprim,phe ny
toin)
,r e
quir
eme nt(
eg,
hemo l
yti
ca nemia,pregnancy)
.
Vit
aminB12 Ca us e s:pe r ni cio usa ne mi a, ma labs o rption ho mo cyst
eine ,  me t
hy lma lo
ni cacid.
(
cobal
amin) (e g , Cr o hndi s e a se),pa nc r e at
ici ns uffic i
enc y, Ne urologicsympt oms :re ve
rsibledeme ntia
,
defc
iency ga st re c tomy ,i ns uffic ienti nt ak e( eg ,v e ganism) , subacutec ombi nedde ge nerati
on( dueto
Di p hy llobo thr ium l a t um( fisht a pe wo rm) . involveme nto fB12i nfat t
ya cidpathwa ysand
Spinocerebellar tract -- Unconcious proprioceptive sensation my eli
n synthe si
s):spinoc er
e bell
artract
, l
ater
al
Lateral Corticospinal tract -- Controls ipsilateral limb fine motion corticospi
na ltract,dorsa lcolumndy s
func t
ion.
Dorsal Column -- White matter in posterior. Made of gracille and cuneatus fasciculi.
Gracile is for lower body sensory and cuneate is upper body sensory. Histori
c a
llydiag no s
edwi t htheSc hill
ingt e
st,
Schilling Test a 4-st
a g
e tes
t tha tdet
e rmi nesifthe ca
u seis
Stage 1- oral B12 and intramuscular B12. dietaryinsufficiencyv sma labsorpti
on.
Urine should have more than 10% of the b12 if patient doesnt have pernicious anemia.
Stage 2- B12 and intrinsic factor. If intrinsic factor fixed it, patient has pernicious anemia. Ane mia2°t oi nsuffici
enti ntakema yt akeseveral
Otherwise there is intestinal malabsorption.
Stage 3- B12 and antibiotics. Identifies bacterial overgrowth.
yearsto develop dueto liver
’sab i
li
tyto s
toreB 1
2
Stage 4- B12 and pancreatic enzymes. Identifies pancreatitis. (asoppos edtof o l
atede fici
enc y)
.
Or
oti
cac
idur
ia I
nabili
tytoc onvertoroticaci
dtoUMP Or o tica c idi nur i ne .
(
denov opy ri
mi dines ynt
hesi
spa t
hwa y
) Tr e atme nt: ur i
di nemo no pho s pha t
eorur idi ne
becauseofde f
ec tinUMPs ynthase. tria ce tatet oby pa ssmut a te de nz y me .
Autosoma lrec
ess i
ve.Prese
ntsinchildr
ena s UWORLD= A 14month boy is evaluated for failure to thrive and developmental delay.
His mother reports that at 12 months he could barely lift his head and had difficulty
f
a i
lur
et othriv
e, developmentaldela
y,and sitting unsupported. He has not started babbling or forming words. He is at the 10th
percentile for height and 5th for weight. Labs show Hb=8.6, MCV=114, Reticulocytes
me gal
oblast
icane mi aref
ract
orytofola
te are 1% and Plasma Ammonia = 42 (normal 40-80). Urine specimens contain large
andB12. Nohy pe r
ammo nemia(vsornit
hine amounts of orotic acid crystals. Supplementation with what substance will benefit
this patient? Uridine. He has hereditary orotic aciduria, resulting in the symptoms seen.
t
ranscarbamy l
asede fici
ency— or ot
icaci
d
withhype ra
mmone mia).
Nonmegal
obl
ast
ic Macrocyt
icanemiai
nwhic
hDNAs y
nthe
sisi
s RBCma c
rocyt
osi
swi
thouthy
per
segme
nte
d
anemi
a nor
ma l. ne
utr
ophil
s.
Caus
e s
:alcoho
lis
m,l
ive
rdi
sea
se.
Di
amond-Bl
ack
fan Ra
pid-onse
tanemiawi
thi
n1 s
tyearofl
ifedueto  
% Hb F(but tot
alHb)
.
a
nemia i
ntr
insicdef
ecti
nery
thr
oidprog
e ni
torcel
ls
. Sho rts
tat
ure,c
rani
ofa
cia
labnormal
it
ies
,and
upperext
remit
yma l
for
mat
ions(t
ri
phalang
eal
thumbs)i
nupt o50%ofca
s e
s.
Prepared for students. Amir Mullick -- YouTube.com/USMLELive Best of luck.
Hemat ol ogyandoncol ogy hematologyandoncol
ogy—P a
thol
ogy S
ect IonI I
I
Intravascular -- Haptoglobin binds free Hb from erythrocytes with high affinity and prevents its oxidative activity. This is then removed by the spleen, so haptoglobin decreases.
Extravascular -- Haptoglobin stays normal since the macrophages that phagocytize RBC will also eat the Hb so theres no free Hb for haptoglobin to bind. LDH rises in this case also.

Normocytic
, Normocytic
,normochr
omi ca nemiasa
reclass
ifie
da sno
nhe mol
yti
corhe moly
tic.
Thehe mo l
yti
c
normochromic anemiasaref
urthe
rclas
sifie
da cc
ordi
ngtothec a
useofthehemoly
sis(
intr
ins
icvse x
tri
nsi
ctothe
anemias RBC)a ndbytheloca
tionoft hehemoly
sis(i
nt r
avas
cul
arvsextr
avas
cular
).Hemo ly
sisc
anleadto
i
ncreas
e si
nLDH,r et
iculocytes
,unconj
ug a
tedbi l
ir
ubi
n,p i
gmente
dg a
llst
ones,andurobi
li
nogen
i
nur i
ne.
I
ntr
avas
cular Findi
ngs: haptogl
obin, sc
hi s
toc
yte
sonb l
oodsmear
.Charac
teri
st
ichemo gl
obi
nuri
a,
hemolysi
s hemo s
ide
rinuri
a,andurobi
linogeni
nurine.No
tab
lecause
sareme cha
nicalhemol
ysis(
eg,
Spherocytosis, G6PD deficiency, and sickle cell
also have high LDH because its high inside RBC. pros
thet
icvalv
e),par
oxys
ma lnoctur
nalhemogl
obi
nuria
,microa
ngiopat
hichemoly
ticanemia
s.
Extr
avas
cular Findi
ngs
:ma c
rophag
esi
ns pl
eencl
earRBCs.Sphe
roc
ytesi
nperiphe
rals
me ar(
mos
t
hemoly
sis commonlyhere
dit
arys
pherocy
tos
isanda
utoi
mmunehe moly
ticanemia
),nohemo
glo
binur
ia/
hemosi
der
inuri
a.Canpre
sentwit
hurobi
li
nogeni
nurine.

Nonhemol
yti
c,nor
moc
yti
canemi
as
d
eSc
RIP
tIo
n F
Ind
Ing
S
Anemiaofc
hroni
c I
nfamma tion(e g,I L-6 ) he pcidin ir
on, TI BC, f e r
rit
in.
di
seas
e (
rel
e asedbyliver,bindsferroporti
non Normocyti
c,butc anbe comemi cr
ocyt
ic.
Inflammation upregulates hepcidin from the liver. i
ntesti
na lmuc osalc e
llsandma cr
op hages
, Tr
eatment:addressund erl
yingca
useof
Hepcidin blocks the release of iron from iron stores t husi nhibit
ingi r
ont ransport
) r el
easeof i
nfamma t
ion,judicioususeofbl
ood
in macrophages and blocks the production of
transferrin (TIBC), which transports iron in the blood. i ronf r
om ma cropha g
esa nd   i
rona bso
rpti
on t
ransf
usi
on, considererythro
poi
esi
s-
Iron stores are high though, its all stuck in the
macrophages, so ferritin levels are high since it f
romg ut.Associat
edwi t
hcondi ti
onssuch s
timulat
inga ge
nt ssucha sEPO (
eg,i
nchro
nic
stores iron. The body sequesters iron in this way to a schronicinfec t
ions,neoplasti
cdiso r
ders
, ki
dneydisease)
.
save it from foreign invaders. Ferritin is critical since
it is low in IDA. Over a long period of time, ACD chronick i
dne ydisease,anda ut
oimmune
becomes microcytic, hypochromic IDA.
dis
eases(eg,SLE,r heuma t
o i
da rt
hr i
ti
s).
Apl
ast
ica
nemi
a Cause dbyf ail
ur eo rde s
truc t
ionof  
reti
culocytec ount ,  EPO.
A
hema t
opo i
etics te
mc ellsduet o : Pa
nc ytopeniac hara ct
eriz edbya ne mi a,
Ra diat
iona nddr ugs( eg,be nzene, l
e ukopenia,andt hromboc ytope nia(no ttobe
chloramphe nicol,a l
ky l
atinga gent
s, confusedwi tha plasti
cc r
isis,whi chca uses
ant i
me ta
bo lites
) anemi aonly).Nor ma lc e
llmor phology ,
Vir a
lag e
nt s( EBV,HI V, hepa t
it
isvi
ruses
) buthy pocell
ul arbonema r r
owwi thfatty
Fa nconia ne mia( DNAr epairdefec
t i
nfilt
ratio
nA ( drybo nema rrowt ap).
caus i
ngbo nema rrowf ail
ure ;normocytos
is Sy
mpt o
ms :fati
g ue,ma lais
e ,pall
o r,purpur a,
orma cr
oc ytosisma ybes eenonCBC) ; muc osalbleeding ,pe techiae,infe c
tion.
Normal bone marrow has 1:1 ratio of RBC:Fat. alsosho r
ts tature,  i
nc idenc eoftumo r
s/ Tr
e at
me nt:withdr awa lofo ffending
Low RBC = Anemia
Low WBC = Infection leukemi a,c afé-
au-laitspot s
,thumb/ r
adial agent,immunos uppr e ssi
ver egime ns(eg ,
Low Platelet = Petechiae defects anti
thy moc y
teg lobul in,cy cl
ospo ri
ne),bone
Pancytopenia without splenomegaly!
No hematopoietic progenitor cells. Idiopathic( immuneme diated,1°ste
mc ell ma rro
wa l
lograft,RBC/ p
la t
elettransfusion,
defect)
;ma yf ol
lowa c utehe pati
tis bonema rr
ows ti
mul ation( e
g ,GM- CSF) .

UWORLD= A 36yo woman comes to the office due to 2 weeks of fatigue and easy bruising. She also had an upper respiratory tract infection. Physical exam shows no lymphadenopathy or
hepatosplenomegaly. Lab studies are- Hb 7 (L), Erythrocytes 1.8 million (L), MCV 90, Reticulocytes 0.1% (L), platelets 88,000 (L), Leukocytes 2,500 (L), PT 13, PTT 30. A peripheral blood
smear shows normocytic, normochromic RBC. The other cell types are also morphologically normal. Bone marrow biopsy in this patient would reveal what kind of pattern?
Hypocellular marrow filled with fat cells and marrow stroma. This patient has anemia (fatigue, low Hb), thrombocytopenia (easy bruising, low platelets), with profound reticulocytopenia,
morphologically normal cell lines on peripheral smear, and no splenomegaly. The absence of splenomegaly is key in diagnosing aplastic anemia. Bone marrow examination would reveal
marked hypocellularity with fat cells, and aspiration would produce a "dry tap".

A 7yo boy is brought to the office due to fatigue, easy bruising, and frequent epistaxis for 2 weeks. He has no medical problems and takes no medications. His temperature is 98F,
BP 90/60mmHg, pulse is 94/min and respirations are 18/min. Physical exam reveals pallor and multiple ecchymoses. There is no lymphadenopathy or hepatosplenomegaly.
Lab results show Hb 8.2 (L), platelets 40,000 (L), and leukocytes 2,100 (L). A bone marrow biopsy image shows fatty infiltration. What is the most likely diagnosis?
Idiopathic aplastic anemia. The etiology is unknown in most children.
Prepared for students. Amir Mullick -- YouTube.com/USMLELive Best of luck.
S ectIonI I I He mat ol
ogyandonc ol ogy he matol
ogya
ndonc ol
ogy
—Pa
thology

I
ntr
ins
ichemol
yti
canemi
as Means something was wrong with RBC, causing it to lyse. Pigmented gallstones are a complication of any hemolytic anemia.
d
eSc
RIP
tIo
n F
IndI
ngS
Heredi
tar
y Ext
ravascularhe mo l
ysisduetodefectin Sple
no me g
a l
y,ap
las
ticcri
si
s( pa r vov irusB1 9
s
pherocytos
is p
roteinsinteract
ingwi thRBCme mbr ane inf
ection)
.
Now after splenectomy youll see s
kele
to nandpl asmame mbr ane(e
g,a nky
rin, Labs: me anfuorescenceo fRBCsi ne os in
Howell Jolly bodies in peripheral smear.
MCHC-- Mean Corpuscular Hemoglobin b
and3 ,protei
n4 .2,spectr
in)
.Mos t
ly 5-maleimide(EMA)b i
ndingt es t,f ra g i l ityi n
Concentration - ratio of Hb to volume of a
utosoma ldo mina ntinheri
tance
. osmoticfragi
li
tyte
st.Norma lt o  MCVwi th
red cells.
Will show RBC are more likely to lyse in Res
ult
si nsma l
l,roundRBCswi thlesssur
face abundanc eofRBCs . Imagine a normal biconcave RBC, like a partly
full ziploc bag. Fill it up with water til its stiff
hypo-osmolar solutions such as after
the addition of glycerol, so haptoglobin a
reaa ndnoc entralpall
or(MCHC) Treat
me nt:spl
enect
omy . and now you have a spherocyte. This cant
squeeze through spleen sinusoids so it gets
would decrease since Hb will be released. pr
e ma t
ur eremov albyspleen. phagocytized. So remove spleen.
G6PDdefic
ienc
y Defec
ti nG6 PD NADPH reduced Backpa i
n,hemoglobinuri
aafewdaysaf
ter
gl
utathione   RBCs uscept
ibi
lit
ytooxidant oxi
da nts t
ress
.
st
res
s( eg,sulf
adr ugs
,anti
ma l
a r
ial
s,inf
ecti
ons
, Labs:b l
oods mearshowsRBCswi hHe
t inz
fa
vabe a ns)  hemo ly
sis
. bodie sandbitecell
s.
Causesex tr
avascul
a ra
ndintravas
cular “St
res sma kesmee atbit
esoff
avabeanswit
h
hemo l
ysis.X-l
inkedrecess
ive. Hei nzk et
chup.”
Pyr
uvateki
nas
e Aut
osomalr
eces
siv
e.Py ruv
atekinas
edefec
t He
mol
yti
cane
miai
nane
wbo
rn.
de
fic
iency   ATP r ig
idRBCs   ex
trav
ascul
ar
Responsible for PEP to pyruvate conversion in the last step of glycolysis.
he
mo l
ysi
s.I
ncr
easesl e
vel
sof2,3-
BPG Leads to deficiency in ATP and the Na/K pump stops functioning so cells hold on
to too much solute and their volume expands, leading to extravascular hemolysis.
  hemogl
obinaffini
tyf
orO2.
Paroxys
ma l
noc
turnal compl ement-
me diat
edintra
vascularRBCl ys
is Assoc i
atedwitha pla
stica ne mi a .
hemoglobi
nur
ia (
acqui r
edmut at
ioninPI GAg ene   impa i
red Tri
a d:Co ombs⊝ he mol y tica ne mi a ,
In healthy patients, the decay accelerating s
ynthe s
isofGPIa nchorfordecay-a
cc e
lerat
ing panc ytopenia,venoust hr ombos is( e g , Budd-
factor CD55 helps prevent potentially dangerous
effects of complement from acting on naive f
act
o r[DAF/CD5 5]andme mbr a
nei nhibit
or Chi arisyndrome .Renal
) findings would show hemosiderosis due to
excess iron deposition in the proximal tubule.
cells. Complement is part of the innate immune o frea ct
ively
sis[MI RL/CD 59
] thatprot
ects Pat
ie ntsma yreportre
do rpi nkur ine .
system so it cannot distinguish native from
foreign. In PNH, DAF or its anchor protein GPI RB Cme mbranefrom comp l
eme nt
).Acq ui
red Labs:CD5 5/5
9⊝ RBCso nfowc yto me tr y.
are not synthesized. DAF is key for protection.
Patients may develop thrombotic complications mutationina hema t
opoiet
icst
e m cell
. Treatme nt:eculi
zuma b( ta r
g etst ermi na l
like Budd-Chiari due to free Hb from lysed RBC.   i
nc idenceofacuteleukemias
. compl e
me ntp r
otei
nC5 ) .
Si
ckl
ecel
lanemi
a HbSpoi ntmut ati
o ncausesasing l
eami no Comp l
icationsi ns i
cklec elldisease:
A
acidreplac
e men tinβ chain(subst
it
u t
ion A pl
a st
ic crisis(transi
e nt ar
resto f
ofg l
utamica ci
dwi thvali
ne )
.Ca uses erythropoi esisduet op arvovirusB1 9).
extrav
a s
cularandi ntr
avascularhemol y
sis. Auto spl
e ne ctomy( Ho we l
l-J
ollybo di
e s)
Pathogenesi
s:lo
wO2, higha l
ti
tude,oracidosi
s ri
skofi nfe ct
ionb yencaps ulat
ed
preci
pitat
essick
ling(deoxygenatedHbS organi s
ms( eg ,Spne umo niae).
po l
yme r
izes
)   anemi a
,va s
o-occl
usived i
seas
e. Sp l
enici nfa rct/s
eque strati
onc r i
si
s.
Ne wbornsareinitial
lyasymp t
oma ti
cbe causeof Sa l
mo nellaos teomy eliti
s.
HbFa nd HbS. Painfulv as o-oc cl
usivec rise
s:da ctyl
iti
s
He t
erozygot
e s(
sickl
ec el
ltrait
)ha v
ere s
ista
nce (painfuls we llingofha nds/f
ee t
),priapism,
toma lari
a. acutec he stsy ndrome( respir
a t
o r
ydis t
ress
,
8%ofAf r
icanAme ri
cansca r
rya nHbSa ll
el
e. newp ulmo na ryinfil
t r
a t
eso nCXR,c ommo n
Sickl
ec el
lsarecrescent-
shapedRBCsA. causeofde ath),avasc ularnec rosi
s,str
ok e
.
Glut --> Val at 6th position on beta chain. “ Crewc ut”ons kullx-r
ayd uetoma rrow Sicklingi nr ena lme d ulla( Po2)   renal
Valine is hydrophobic, so exposed valine
at one side will stick to valine at neighboring expansionfrom e ryt
hropoiesis(
alsoseenin papillaryne crosis   he ma t
uria .
Hb and polymerize. Offspring of carrier thalas
semia s
). Di
agno sis
:he mog lobine lec t
ropho res
is.
parents has 25% chance of being affected,
50% heterozygous; 75% receive mutant allele. Tr
eatme nt:hydr o xyurea(HbF) ,hy dr
ation.
HbCdi
sea
se Glut
amicaci
d–t
o-l
yCine(l
ysi
ne)mut
ati
onin Pa
tie
ntswit
hHbSC( 1ofeac
hmut a
ntge
ne)ha
ve
β-g
lobi
n.Caus
esext
rava
scul
arhemol
ysi
s. mil
derdis
e a
set
hanHbSSpa t
ients
.
Bl
oodsme ari
nhomozyg
otes
:hemo gl
obi
n
Crys
tal
sinsi
deRBCs,t
arg
etcells
.

Acute chest syndrome is most common cause of death, usually due to infection induced inflammation that precipitates vaso-occlussion. Note fever, cough, extreme pain, dyspnea, and eventually
hypoxia and death. The intra and extravascular hemolysis results in increased indirect bilirubin and LDH, while decreasing haptoglobin, which binds circulating Hb and reduces renal excretion of
free Hb.
Prepared for students. Amir Mullick -- YouTube.com/USMLELive Best of luck.
Hemat ol ogyandoncol ogy hematologyandoncol
ogy—P a
thol
ogy S
ect IonI I
I

Ext
rins
ichemol
yti
canemi
as
d
eSc
RIP
tIo
n F
Ind
Ing
S
Autoi
mmune Wa m(
r IgG)—c hronicanemi aseeninSLE Aut oimmunehe mo l
yti
ca nemiasa r
eus
uall
y
hemolyt
ica
nemi
a andCLLa ndwi thcertai
ndr ugs(eg, Co ombs⊕.
A
α-me t
hy l
dopa)( wa
“ rm we a
therisGr e
a t
”). Di rectCoo mbstest—anti
-Igantibody(
Coombs
Cold(IgMa ndc omp l
eme nt)
—a cute r
e agent)addedtopatie
nt ’
sRBCs .RBCs
anemi atr
iggeredbyc o l
d;seeninCLL, agglutina
teifRBCsa rec oat
edwi t
hIg.
My coplas
mapne umo niaeinfect
ions,and Indi r
ec tCoombste s
t—no rmalRBCsa ddedto
infe
ctiousMo nonucleosi
s(“coldwe at
heris pati
e nt’
sser
um.I fser
um ha santi-
RBCsurfa
ce
MMMi ser
ab l
e ”
).RBCa gglut
inatesA ma y I
g ,RBCsa ggl
uti
na t
ewhe nCoo mbsr
eagent
causepa i
nful,bluefingersandtoeswithc ol
d added .
exposure.
Ma nywa rma ndc oldAI HAsa reidi
opathic
.

P
ati
entc
omp
one
nt Re
age
nt(
s) Re
sul
t Resul
t
(
agg
luti
nat
ion) (
noa
gglu
tin
ati
on)
s
omb

Uses patient's RBC.


Co

Addition of Coombs reagent


t

will cause agglutination if the


c
e

RBC have anti-RBC Ab.


r
Di

R
BCs+
/–a
nti
-R
BCA
b A
nti
-h
u mangl
obul
in R
esul
t Res
ult
(
Coombsre
agen
t) A
nti
-RB
CA bpr
ese
nt A
nti
-RB
CAbabse
nt
tCoombs

Uses patient's serum.


Addition of donor blood and Don
orb
loo
d
Coombs reagent will cause
c
e

agglutination if serum had


dr
i

anti-RBC Ab.
n
I

Pa
ti
ents
eru
m+ /
– A
nti
-h
u mangl
obul
in Re
sul
t R
esul
t
a
nti
-d
onorRBCAb (
Coombsre
agen
t) A
nti
-d
ono
rRBCA
bpr
ese
ntA
nti
-do
norRBCAba
bse
nt

Micr
oangi
opat
hic Pa
thoge
nesis
:RBCsa r
eda magedwhenpass
ing Sc
hist
ocyte s(
eg,“
he l
me tce
lls
”)ar
eseenon
a
nemia t
hroughobst
ruct
edornarrowedvess
ell
umina. per
ipher
a lbl
oodsme arduetome c
hanic
al
Se
eninDI C,TTP/HUS, SLE,HELLP des
truc
tion(schi
sto=t osp
lit
)ofRBCs.
s
yndro
me ,hype
rte
nsiv
ee merg
e nc
y.
Macr
oangi
opat
hic Pr
osthet
ichear
tval
vesanda
ort
ics
tenos
ismay Schi
st
ocy
teso
npe
riphe
ralbl
oods
mea
r.
a
nemia a
lsoc a
usehemoly
tica
nemia2°t
ome chani
cal
des
tructi
onofRBCs.
I
nfec
tions d
est
ruc
tionofRBCs(
eg,ma
lar
ia,Ba
bes
ia)
.

UWORLD= A 60yo African-American female is evaluated for fatigue. Past surgical history includes mitral valve repair with a mechanical valve due to severe mitral insufficiency. She has a 30
pack year history of smoking and occasionally drinks alcohol. Examination shows pallor. Lab studies show Hb 9 (L), reticulocytes 6.8% (H), platelets 218,000 (n), and leukocyte 4,500 (n). Serum
chemistries are normal. Peripheral smear shows burr and helmet cells with RBC fragments. What most likely caused her condition? Mechanical trauma. RBC fragments, burr cells, and helmet
cells are consistent with traumatic hemolysis, which can result from microangiopathic hemolytic anemia or mechanical damage. Prosthetic valves produce excessive shear and turbulence.
Prepared for students. Amir Mullick -- YouTube.com/USMLELive Best of luck.
S ectIonI I I He mat ol
ogyandonc ol ogy he matol
ogya
ndonc ol
ogy
—Pa
thology

I
nter
pret
ati
onofi
rons
tudi
es
I
ron Chr
onic Pr
egnancy
/
defic
iency di
seas
e Hemoc
hroma
tos
is OCPuse
S
erum i
ron —
T
rans
fer
ri
norT
IBC a

F
err
it
in —
%tra
nsf
erri
nsatur
ati
on —
(
ser
um i
ron/TI
BC)
=1 °d is
tur
bance .
Transfer
rin—transpo rt
sir
oni nbl
o od.
TIBC—i ndir
ectl
yme as
urest
ransf
errin.
Ferr
itin
—1 °ironstoragepr
o t
einofbody.
aEvoluti
onaryreas
o ning—pathogensusecir
culat
ingir
ontot
hri
ve.Thebodyhasa
dapt
edas
yst
emi
nwhi
chi
roni
sst
ore
d
withinthec e
llsofthebodya ndprevent
spathogensf
romac
quir
ingcir
cul
ati
ngir
on.

L
eukopeni
as
c
ellt
yPe c
ellc
oUn
t c
aUS
eS
Neut
rope
nia Abs
olut
eneut
rophi
lcount<1500c
ell
smm3
/ Se
psi
s/
post
inf
ect
ion,dr
ugs(i
ncl
uding
Sev
erei
nfe
cti
onsty
picalwhe
n<500c el
ls
/mm3 c
hemother
apy)
,apl
ast
icane
mia,SLE,
r
adi
ati
on
L
ymphopeni
a Abs
olutel
ymphoc
ytecount<1500c
ell
smm3
/ HIV,DiGeorgesyndrome,SCID,SLE,
(
< 3000cel
ls
/mm³inchil
dre
n) c
orti
cost
eroi
dsa
,r a
diat
ion,s
epsi
s,po
stope
rat
ive
Eos
inopeni
a Abs
olut
eeos
ino
phi
lcount<30c
ell
smm3
/ Cus
hings
yndr
ome
,cor
ti
cos
ter
oi a
ds
a
Cor
ti
cos
ter
oidsc
aus
ene
utr
ophi
li
a,de
spi
tec
aus
inge
osi
nope
niaa
ndl
ymphope
nia
.Cor
ti
cos
ter
oids ac
tiv
ati
onofne ut
rophi
l
a
dhes
ionmolec
ule
s,i
mpai
ringmigr
ati
onoutofthevas
cula
turetos
it
esofi
nfa
mma
ti
on.
Inc
ont
ras
t,c
orti
cos
ter
oidsseques
ter
e
osi
nophi
lsi
nlymphnode
sa ndc
auseapop
tos
isofly
mp hocy
tes.

Neut
rophi
llef
tshi
ft  
ne utr
ophilprecursor
s,sucha sbandc el
ls Ale
fts
hif
tisashi
fttoamo r
eimma
tur
ece
lli
n
A metamyelocyte is a cell undergoing andme t
amy elocyt
es,inpe r
iphera
lblood. t
hema t
urat
ionproce
ss.
granulopoiesis, derived from a myelocyte, Us ual
lyseenwi thneutrophili
ainthea cute
and leading to a band cell. It is
characterized by the appearance of a bent r
e sponsetoinfecti
onori nfamma t
ion.Ca l
led
nucleus, cytoplasmic granules, and the
absence of visible nucleoli. l
e ukoerythroblast
icreactonwhe
i nl ef
tshif
tis
s
e enwi t
hi mma tureRBCs . Occurswithsever
e
ane mia(physi
ologicresponse)orma rr
ow
r
e sponse(eg,fib
rosi
s,tumo rtaki
ngups p a
cein
ma rrow)
.

Lab characteristics of coagulopathies!


PT is only elevated in DIC and Warfarin use (vitamin K deficiency).
PTT is elevated in hemophilias, DIC, and heparin use.
Platelets are only low in DIC and immune thrombocytopenia.
Bleeding time is increased in vWFD, DIC, uremic platelet dysfunction, and immune thrombocytopenia.

So everything is altered in DIC, where loss of factors elevates both PT and PTT, and you use up platelets so bleeding time increases.
Everything is normal in hemophilia except PTT since either factor 8 or 9 will be lost.
Everything is normal in vWFD except bleeding time is increased since platelets will not adhere and aggregate well. PTT may be high due to loss of factor 8.
Heparin administration only raises PTT since factors 2 and 10 are not working, but if platelets are not normal then suspect HIT.
Platelet disorders all only effect platelet count and increase bleeding time, except in Bernard and glanzmann, where platelet count is normal but they do not adhere and aggregate.

A young girl with elevated PTT and bleeding time most likely has vWF disease because its autosomal dominant, whereas a young man with elevated PTT and family history of men with
bleeding or hemarthroses most likely has hemophilia, since it is X-linked recessive.
Prepared for students. Amir Mullick -- YouTube.com/USMLELive Best of luck.
Hemat ol ogyandoncol ogy hematologyandoncol
ogy—P a
thol
ogy S
ect IonI I
I

Hemesynthesi
s, Theporphyri
asareher
edit
aryorac
qui
redcondi
tionsofdef
ect
ivehe
mes y
nthe
sistha
tlea
dtothe
por
phyri
as,andlead ac
cumul at
ionofhemep r
ecurs
ors
.Le
adinhibi
tsspeci
fice
nzymesnee
dedinhe mesynt
hes
is
,
poi
soni
ng l
eadi
ngt oasimil
arcondi
ti
on.
c
ondI
tI
on a
FFe
cte
denZ
yme a
ccU
mUl
ate
dSU
BSt
Rat
e P
ReS
ent
IngS
ymP
tomS
Fer
roc
hel
atas
eand Pr
otopor
phy
rin,
ALA Microcyt
ica ne mia(b
asophil
icsti
ppl
ingin
L
eadpoi
soni
ng ALAdehydr
ata
se (
blood) peri
pheralsme a
rA,r i
ngedsider
oblast
sin
A
An 18 month old boy is brought to the office due to language bonema rrow) ,GIandkidneydisea
se.
regression. He said several words at his 1-year appointment but
no longer speaks any words. His moods are also unpredictable Chil
dren—e xposuret
oleadpa i
nt me ntal
and he throws tantrums. The parents tried to bring him earlier but
they live in an impoverished part of the city and are poor. On
det
e r
ior
ation.
physical exam, the boy is quiet and well. Hb is 9 g/dL. Which of Adult
s—e nvironme nt
alexposure(
eg,batt
eri
es,
the following enzymes is most likely inhibited in this patient?
ALA Dehydratase. Lead toxicity is prevalent among impoverished ammuni ti
on) he a
dache,me moryloss
,
children in urban housing built before 1978. Protoporphyrin 9 cant
incorporate Fe2+ so it incorporates zinc; high zinc protoporphyin.
demy e
li
na ti
o n.

Acut
eint
ermi
tt
ent Porphob i
li
noge
n Po
rpho
bil
ino
gen,
ALA Sy
mpt oms(5P’ s)
:
por
phyr
ia deami nase
, Pa i
nfulabdome n
previousl
yknownas Po r
twine –color
edur ine
uroporphyri
nogenI Po l
yneuropathy No photosensitivity.

synthase(a
utos
omal Psychologicaldi
stur
ba nces
domi nantmutat
ion
) Precipi
tatedbydr ugs(eg,cytochr omeP- 4 50
inducers)
,a l
cohol
, s
tarvati
on
Tr
eatment:he minandg lucose.
Porphy
riac
utanea Uro
po r
phyri
noge
n Ur
oporphy
rin(t
ea- Bli
st
eri
ngc ut
a neouspho t
osensi
tivi
tyand
t
arda dec
a r
boxy
las
e c
olor
edurine
) hyperpigment a
tionB.
B
(
autos
omaldomina
nt Mostcommonpo rphyr
ia.Exacerbat
edwith
mutati
on) al
coholc onsumpt ion.
Hepatitis C is usually transmitted parenterally, so look out for Ass
ociatedwithhe pa t
it
isC.
patients who are healthcare workes or IV drug users. The
patient will present with scleral icterus and jaundice plus a
Trea
tme nt:phlebotomy,suna v
oidanc e
,
pruritic rash of PCT that worsens upon exposure to sunlight. anti
ma lar
ials(eg,hydroxyc
hloroquine)
.
Patients who received blood transfusions before 1992 may
also have HepC because blood was not screened for it before.

L
oca
ti
on I
nte
rme
dia
tes E
nzy
mes Di
sea
ses

Gl
yci
ne+s
ucc
iny
l-
CoA

Mi
toc
hon
dri
a – Gl
uco
se,
hemi
n Ami
nol
evul
i
nicac
id
B
6 S
ide
rob
las
ti
can
emi
a(X
-li
nke
d)
UWORLD - The question stem had a patient with s
ynt
has
e(r
at
e-l
i
miti
ngst
ep)
latent TB taking isoniazid and presenting with a A
minol
evu
li
ni
cacid
history of progressive fatigue and conjunctival
pallor. CBC shows a Hb of 9 and MCV of 72, which A
minol
evu
li
ni
caci
d
hints at microcytic anemia. Bone marrow aspirate L
eadp
ois
oni
ng
under Prussian blue stain showing ringed sideroblasts
d
ehydr
ata
se
as iron is transported to developing erythrocytes that Porph
obi
li
noge
n
cannot form heme, so its granules accumulate around P
orpho
bil
i
nogen
the nucleus. The patient has sideroblastic anemia due d
eamin
ase A
cut
ein
ter
mit
te
ntp
orp
hyr
i
a
to isoniazid-induced B6 deficiency; ALA Synthase. H ydr
oxy
methy
lbi
l
ane
C
ytop
las
m
Ur
opo
rph
yri
nog
enI
I
I
Ur
opor
phyr
in
ogen
P
orp
hyr
i
acu
tan
eat
ar
da
dec
arbo
xyl
ase
Co
pro
por
phy
ri
nog
enI
I
I

P
rot
opo
rph
yri
n
Mi
toc
hon
dri
a F
e2+
F
err
och
ela
tas
e L
eadp
ois
oni
ng
He
me
22 yo woman presents with a 5-day history of nausea, constipation, and severe abdominal pain. She
↓h
eme→ ↑A
LAs
ynt
has
eac
ti
vi
tyalso reports anxiety, difficulty concentrating, and tingling of the limbs. The patient has had many similar
↑h
eme→ ↓A
LAs
ynt
has
eac
ti
vi
ty events in the past. No medications, tobacco, alcohol, or drugs. Patient is restricting her diet to lose
weight. The patient receives IV heme and rapidly feels better. The improvement in symptoms is due to
downregulation of which enzyme? ALA Synthase.
Prepared for students. Amir Mullick -- YouTube.com/USMLELive Best of luck.
S ectIonI I I He mat ol
ogyandonc ol ogy he matol
ogya
ndonc ol
ogy
—Pa
thology

I
ronpoi
soni
ng
Ac
ute Chr
oni
c
F
Ind
Ing
S Hi
ghmor t
ali
tyra
teass
oci
ate
dwi t
hacci
dent
al Se
eninpati
entswit
h1 °(her
edit
ary)or2°(
eg,
i
nges
ti
onb ychi
ldr
en(ad
ultir
ontab
let
smay c
hroni
cbloodtra
nsfusi
onsfort
ha l
asse
miaor
l
ookl
ikecandy)
. s
ick
lecel
ldis
ease
)he mochromatosi
s.
me
cha
nIS
m Cel
ldeat
hduetofo
rmat
ionoffre
eradi
cal
sand
p
erox
idat
ionofmemb
raneli
pids.
S
ymP
tomS
/SI
gnS Abdominalpai
n,vomit
ing,GIble
edi ng. Ar
thr
opat
hy,
c i
rr
hosi
s,ca
rdi
omy
opa
thy
,di
abe
tes
Radi
opaquepills
eenonx -
ray
.Ma yp r
ogre
ssto me l
li
tus
,hypogo
nadis
m.
ani
ong apme t
abol
icaci
dosi
sandmul ti
organ
fa
il
ure
. Leadstos
carr
ingwithGIo bst
ruct
ion.
t
Rea
tme
nt Chel
ati
on(eg
,def
ero
xami
ne,d
efe
ras
ir
ox)
, Phle
botomy(pa
ti
ent
swi
tho
uta
nemi
a)o
r
ga
str
icla
vage
. chel
ati
on.

Coagul
ati
ondi
sor
der
s PT—t est
sfunc ti
ono fc ommona nde xtr
insi
cpa t
hwa y(fa
ctor
sI,II,V, VII,a
ndX) .Defect PT
(PlayTenni soutside[ ext
ri
nsicpathway])
.
INR( i
nternationalno r
ma l
izedrati
o)—c al
culat
edfrom PT. 1=no rma l,>1=pr o l
onged.Most
commo ntestuse dtof oll
owpa ti
entsonwa rf
ari
n, whichprol
ong sINR.
PTT—t estsfunc t
iono fcommo na ndintri
nsicpat
hwa y(allf
act
o r
se xceptVIIandXI I
I)
.De f
ect
PTT( PlayTa b leTennisinsi
de).
Co a
gulati
o ndisorde rscanbeduet ocl
otti
ngf a
ctordefic
ienci
esora cquire
di nhi
bit
ors
.Di a
gnose
d
withami xings t
udy ,inwhichnor malplasmaisadd e
dt opati
ent’splasma.Clot
tingfact
or
defic
ienciesshouldc orr
ect(
thePTo rPTTr e
turnstowi t
hinthea ppropri
atenormalrange)
,
whe r
easfactorinhi bitor
swillnotcorre
c t
.
d
ISo
Rde
R P
t P
tt me
cha
nIS
man
dco
mme
ntS
Hemophi
li
aA,
B,orC — Intr
insicpa t
hwaycoagula t
iondefect(PTT) .
A A: defic i
encyoff
ac t
orVI II
;X-li
nkedrecess
ive
.Most common.
B:d eficiencyoff
actorI X;X-l
inkedreces
si
ve.Christmas Disease.
C: de fici
encyoffact
o rXI;autos
oma lre
cess
ive.
He mo rr
ha g einhemo phili
a—he marthros
es(bl
eedingi ntoj oints,eg,kneeA) ,
e
asybr ui si
ng,bl
eedinga ft
ertra
umaors urger
y(eg,de ntalpr oce d
ures
).
Treatme nt :des
mo pr
essin+f act
orVI I
Iconcentr
ate(A) ; f
ac t
o rI Xc onc
e nt
rat
e
(
B);facto rXIconcentrate(C).

Vi
tami
nKdefic
ienc
y Gener
alcoagul
ati
ondef
ect
.Bl
eedi
ngti
menormal
.
a
c t
iv
ityoff
act
orsII
,VI
I,I
X,X,pr
ote
inC,p
rote
inS.
PT is better for testing for vitamin K deficiency because the entire extrinsic pathway is deficient, as well as the common pathway.

Three stages of coagulation. First, vasoconstriction to decrease blood flow. Then, primary hemostasis where platelets form a plug.
Lastly, secondary hemostasis where clotting factors form a fibrin mesh network.

PT checks 1, 2, 5, 7, and 10. This is common pathway plus factor 7 from extrinsic pathway.
PTT checks all factors except 7 and 13, so it covers common and intrinsic pathway, which is 12, 11, 9, and 8.

1, 2, 5, and 10 are from common pathway.

UWORLD= A 5yo boy has prolonged epistaxis that began an hour after being elbowed in the face during a basketball game. He
has a history of hemarthrosis in the right knee that developed after playing soccer with friends. His maternal uncle also has a history
of hemarthrosis and prolonged bleeding after dental procedures. Vital signs are normal. The patient receives desmopressin, which
effectively stops the bleeding. What is the most likely therapeutic mechanism of desmopressin for this patient?
Increase circulating factor 8. This patient's bleeding history, response to desmopressin, and family history of a maternal uncle with
hemarthrosis is consistent with hemophilia A. Factor 8 is produced by liver sinusoidal endothelial cells, and despmopressin
significantly increases circulating factor 8 levels and stimulates vWF secretion from endothelial cells. Desmopressin (DDAVP) also
acts as an ADH analogue by binding to V2 receptors in the renal tubular cells, leading to increased aquaporin channels, increased
water reabsorption, and decreased urine output, which all helps patients with central diabetes insipidus and nocturnal enuresis.
Prepared for students. Amir Mullick -- YouTube.com/USMLELive Best of luck.
Hemat ol ogyandoncol ogy hematologyandoncol
ogy—P a
thol
ogy S
ect IonI I
I
Compared to clotting factor disorders- platelet disorders give you prolonged bleeding and mucosal bleeding because without platelets you cannot make the fibrin mesh, while coagulation factor
disorders have delayed bleeds because the initial platelet plug forms well but bleeding occurs later in time due to the immature and weak structure of the clot due to lack of appropriate clotting
factors to stabilize it. Bleeds will also be deeper such as into joint spaces.
Coagulation disorder = Delayed Bleeding Platelet disorder = Prolonged Bleeding

Pl
atel
etdi
sor
der
s Allp
latele
tdi
sor
der
shave b l
eedi
ngtime(BT),
muc ousme mbranebl
eedi
ng ,a
nd
microhemor
rhage
s(eg,pet
echi
ae,e
pist
axi
s)
.Pl
ate
letcount(PC)isus
uall
ylow,butma
ybe
norma li
nqual
it
ati
vediso
rder
s.
d
ISo
Rde
R P
c B
t n
ote
S
Bernar
d-Soul
i
er –/ Def
ectinadhe
sio
n. Gp I
b pl
ate
let
-t
o-v
WFadhe
sio
n.
syndr
ome La
bs:abno
rmalris
toc
eti
ntes
t,l
arg
ep l
at
ele
ts.
Gl
anzmann – Def
ecti
na g
gre
gati
on. GpII
b/I
IIa(i nt
egrnαI
i I
bβ) pl
3 ate l
e t-to-pl atelet
t
hrombas
the
nia a
ggreg
ati
onanddefe
cti
vepl
ate
letpl
ugformati
on. Normal response to ristocetin. Fibrinogen
normally binds Gp2b/3a and links platelets,
La
bs:bl
oodsmears
howsnoplat
eletc
lumping. since there is available vWF and Gp1b

I
mmune De s
tructi
ono fplat
ele
tsins pl
een. Anti-
GpIIb/
II
I aanti
bodies s p le
nic
thr
omboc
ytopeni
a ma crophagesphagocy t
osep l
at
e l
ets.Maybeidio pat
hicor2 °toautoimmune
dis
or der
s(eg,SLE),v i
ralil
lness(eg,HIV,HCV) ,ma lig
nanc y(e
g ,CLL),or
drugr eac
tio
ns .
Labs: me gakaryocyt
e sonbonema r
rowbiopsy
, pl at
e l
etcount.
Treat
me nt:st
eroi
ds,IVI G,rit
uxima b,TPO receptoragonis
ts(eg,elt
rombopag,
romiplost
im),orsplenectomyf orrefr
act
oryITP.
Thrombot i
c Disorder
sov erlapsignificantlyins ympto ma t
ology.
thromboc ytopenic Pat
hophy siol
ogy :
purpuraand TTP: inhibitionord efic
ienc yofADAMTS 13(av WFme ta
lloprot
ease)
hemol yt
ic-uremic  de g
rada t
ionofv WFmul t
ime r
s l ar
g evWFmul ti
me rs pl a
te l
et
syndrome adhesiona nda ggre gation(mi crot
hro mbiforma ti
on)
.
HUS:c ommo nlyc ause dbys higa-l
iket o
xinf ro
m EHEC( s
erotypeO1 57:H7)
infe
ction. Atypicalform( aHUS)i sc ausedbyc omple mentgenemut a
tions
orautoimmuner espons e.
A 46yo woman comes to the ED due to 4 days of intermittent fever, Pr
e s
e nt
ati
on: tria
doft hromboc ytopenia,mi cr
oa ngi
opathichemo lyti
canemi a,
abdominal pain, and vomiting. For the past 2 days she has had
decreased urinary output, skin rash, and progressive lethargy. acutekidne yinjury.Als o:
Her T=101F, BP=130/80, and P=100 There is scattered petechial
rash, facial puffiness, and 1+bilateral pedal edema on physical exam.
TTP: pe ntad(tri
ad+f e ver+ne urologicsympt oms).
Lab studies show Hb=8.9 with elevated reticulocyte count and a platelet HUS:hi st
o r
yofb lo odydi arr
he a.
count of 26,000. Bleeding time is prolonged and PT and aPTT are normal.
Peripheral blood smear shows shistocytes and reduced platelets with Epidemiology :
presence of giant forms. BUN=46 and creatinine=2.3 with UA positive for
proteinuria and hematuria. What would you see on renal biopsy?
TTP ,typicall
yi nfe ma les
.
Platelet-rich thrombi in glomeruli and arterioles. Her pentad of fever, HUS, ty
pic al
lyinc hi l
dren.
neurologic lethargy, renal failure, anemia, and thrombocytopenia in the
setting of gastrointestinal illness most likely hints at TTP-HUS. Labs:p l
a t
eletcount ,he mo lyt
ica nemi a(eg,schist
ocyt
es, LDH) .Norma lPT/
PTThe lpsdi s
ti
ngui shHUSa ndTTP( coagul
a ti
onpathwa yisnota c
tiv
ate d
)
f
rom DI C( coa gulat
io npa thwa yisactivated)
.
Tr
e atment:pla s
ma phe resi
s ,s
teroids,r
ituxima b.
HUS == ART. Anemia, Renal, and Thrombocytopenia + Bloody Diarrhea in kids.
TTP == FuN ART, Fever, Neurologic, Anemia, Renal, and Thrombocytopenia

UWORLD= A 7yo boy comes to the office for follow-up. A week ago, the patient began having episodes of bloody diarrhea, and these have since resolved. Currently, lab studies show acute
renal failure. A CBC shows anemia and thrombocytopenia. Coagulation studies are normal. His peripheral blood smear shows lots of schistocytes. What is the most likely cause of his anemia?
Microangiopathic hemolytic anemia. The clinical presentation of ART (anemia, renal failure, and thrombocytopenia) preceded by a case of childhood enterohemorrhagic diarrhea is consistent
with Hemolytic Uremic Syndrome. The schistocytes and few platelets in the blood smear are diagnostic of a traumatic mechanism or mechanical damage.

A 35yo woman comes to the hospital due to numbness of the left arm and face. The patient has no weakness but has had several days of generalized headache, dyspnea on exertion, and easy
fatigability. She has a history of well-controlled asthma. Temperature is 99.8F, BP is 110/60 mmHg, and pulse is 80/min. Light touch sensation is decreased in the left upper extremity and the
lower left face. Strength and reflexes are normal. Cardiopulmonary examinations are unremarkable. She has no skin rash. Lab results show Hb 8.6 (L), platelets 24,000 (L), BUN 32 (H), and
creatinine 1.9 (H). PT and PTT are normal. Peripheral blood smear shows schistocytes. Urinalysis is positive for hematuria and proteinuria. What is the most likely primary pathogenesis?
Impaired cleavage of vWF. Thrombotic thrombocytopenic purpura presents with fever, neurologic symptoms (decreased light touch sensation in this patient), anemia (fatigue), renal failure (as
seen in her labs as well as urinalysis), and thrombocytopenia (noted by her low platelets). The anemia is hemolytic anemia, so LDH would be high and haptoglobin would be low since its binding
all the free Hb from dead RBCs. Her schistocytes further confirm the diagnosis.

A 32yo woman reports fatigue and malaise over the last several days. The patient has not had any sick contacts. She does not appear to be in acute distress. After her initial evaluation, TTP is
suspected. What is essential in making the diagnosis? Abnormal neurological exam, elevated BP, elevated liver enzymes, fever, hemolytic anemia, renal failure.
Hemolytic anemia. TTP is one of the primary thrombotic microangiopathies and a hallmarks of the diagnosis are nonimmune hemolytic anemia with schistocytes and thrombocytopenia without
any other clear cause. Plasma exchange is life-saving and should be considered.

A 22yo woman comes to the ED due to a nosebleed. She had a similar episode yesterday, but the bleeding stopped with prolonged local pressure. On review of systems, the patient has also had
easy bruising for the past several months. She has no significant past medical history and takes no medications. On physical exam, her heart and lungs appear normal. The liver span is 8CM and
the spleen is not palpable. There are scattered ecchymoses over her arms and legs. Labs are perfectly fine except her platelet count is 9,000 which is low but the question stem gave all her labs
so it would help to remember what is normal. HIV and hepatitis C are negative. What is the most likely primary mechanism of her condition?
Immune destruction of platelets. The patient has recurrent epistaxis, ecchymoses, and marked thrombocytopenia but everything else is unremarkable. This is primary immune thrombocytopenia,
characterized by autoimmune destruction of GP2b3a. In children this is typically acute and self-limited but in adults it is insidious.
Prepared for students. Amir Mullick -- YouTube.com/USMLELive Best of luck.
S ectIonI I I He mat ol
ogyandonc ol ogy he matol
ogya
ndonc ol
ogy
—Pa
thology

Mi
xedpl
atel
etandc
oagul
ati
ondi
sor
der
s
d
ISo
Rde
R P
c B
t P
t P
tt n
ote
S
v
onWil
lebr
and — — —/ I
ntri
nsicpathwaycoagulatio
nde fe
ct:v WF
dis
eas
e   PTT( vWFa ctstoc a
rry/
prot
ectfact
or
UWORLD= A 24yo woman is being evaluated for chronic fatigue. She
has a history of heavy menstrual periods since menarche and also V I
II
).
recalls frequent nosebleeds as a child. Her past medical history is De fectinplat
ele
tp l
ugforma t
ion: v WF
otherwise insignificant and she takes no medications. Lab shows a
Hb of 9.2, mcv of 72, and low ferritin. Most likely diagnosis? vWD.  
de f
ectinplat
el
et-
to-vWFa dhesion.
This patient's microcytic anemia with low ferritin is characterisitc of
iron deficiency anemia. Her history of menorrhagia and epistaxis Autos
o maldomin a
n t
.Mi ldbutmo stcommo n
suggest vWD as the most likely etiology of her anemia. Due to
defective platelet binding and aggregation, patients may also have
i
n heri
tedbleedi
ng dis
order.No pl
atel
et
GIT bleeding, leading to iron deficiency anemia. aggregat
ionwithris
toceti
nc of
actoras
say.
Treatment:des
mo pres
sin,whichrelea
ses
vWFs t
oredinendotheli
um.
Di
sseminated Wid es
preada ct
ivat
iono fclott
ing   defic
ienc
y
i
ntr
avasc
ul ar inclotti
ngfa c
tors   b
leedings t
ate
.
c
oagulat
ion UWORLD= A 50yo woman with a history of SLE is admitted with Ca uses:Sna kebit
es,Sepsis(gram ⊝),
fever, chills, and burning pain when urinating. She quickly becomes
hypotensive despite aggressive resuscitation. Urine and blood cultures T rauma ,Obs t
etri
cc ompl i
cati
ons,acute
grow Gram (-) rods. One hour after admission she starts bleeding from P an cr
eati
tis
, Ma l
ignancy,N ephrot
ic
venipuncture sites. Lab results are Hb 9 (L), platelets 68,000 (L),
leukocytes 24,500, PT 23 (H), PTT 60 (H), and plasma fibrinogen syndrome ,Transf
usion(SSTOPMa king
100 (L). What is the most likely diagnosis? DIC
This patient has a UTI complicated by septicemia, as documented by Ne wThr ombi )
.
the urine and blood cultures positive for gram (-) rodes. DIC should L abs:schis
tocytes
, fibrinde gra
dati
on
always be suspected in any sick patient with elevated PT and PTT
along with low platelet and fibrinogen levels as well as fibrin products(d-dimers)
, fibrinogen, fac t
orsV
degradation products called D-dimers.
andVI II.
vWF Disease is the most common inherited bleeding disease. Factor V leiden is the most common inherited hypercoagulability.

Her
edi
tar
ythr
ombos
iss
yndr
omesl
eadi
ngt
ohyper
coagul
abi
li
ty
d
ISe
aSe d
eSc
RIP
tIo
n
Ant
ithrombi
n I
nheri
teddefici
encyofanti
thr
ombin:
ha snodi re c teffe c to nt hePT,PTT,o rthr
ombint
imebut
de
fici
ency di
minishestheincre
aseinPTTfoll
owi nghe pa rina dmi ni str
ati on.
Canalsobea cq
uired:r
enalfa
il
ure
/nephr otics y ndr ome a nti
thr ombi nl os
sinuri
ne
  inhib
iti
ono ffact
orsI
IaandXa. Inheritance is usually autosomal dominant.
F
act
orVL
eiden Produc ti
ono fmutantfa
ctorV( gua
nine   adeni neDNApo intmut ati
o n   Ar g 506Gl nmut at ion
nea rthecleava
g esi
te
)thatisre
sis
tanttode grada tio nbya ctiva tedp roteinC.Mo s tc ommon
caus eofinheri
tedhyperc
oa gul
abili
tyi
nCa uc asia ns .Compl i
c at ionsi nc ludeDVT,c ere bralv ein
thrombo sis
,recurr
entpre
g nancyloss
.The DVT are often the source of pulmonary emboli. This is autosomal dominant.
Protei
nCorS abi
lit
ytoinac
tiva
tefa
ctor
sVaa ndVII
Ia.r i
skofthr
omboti
csk
inne cro
siswi
thhemor
rhage
defic
iency af
teradmini
st
rati
onofwarfa
rin.I
fthi
soc c
urs,t
hinkprot
ei
nCd efic
iency
.Toget
her
,pr
ote
inC
Cancels
,andprote
inSStops,coa
gula
tion.
Prot
hrombingene Muta
ti n3′
oni unt
rans
lat
edr
egi
on p
rod
uct
ionofpr
othr
omb
in pl
asmal
eve
lsa
ndv
eno
us
mutati
on c
lot
s.

Glanzmann thrombosthenia
Prepared for students. Amir Mullick -- YouTube.com/USMLELive Best of luck.
Hemat ol ogyandoncol ogy hematologyandoncol
ogy—P a
thol
ogy S
ect IonI I
I

Bl
oodt
rans
fus
iont
her
apy
c
omP
one
nt d
oSa
gee
FFe
ct c
lIn
Ica
lUS
e
Pac
kedRBCs Hba
ndO2c
arr
yingc
apa
cit
y Ac
utebl
oodl
oss
,se
ver
eane
mia
Pl
atel
ets pl
at
ele
tcount(∼5 mm3/
000/ uni
t) St
opsigni
fic
antbl
eedi
ng(t
hromboc
yto
peni
a,
qua
lit
ati
veplat
ele
tdef
ect
s)
F
res
hfr
oz e
n coagulati
onfact
orlev
els
;FFPconta
insall Ci
rr
hos
is
,imme
dia
tea
nti
coa
gul
ati
onr
eve
rsa
l
pla
sma/prothrombin coagula
tionfact
orsa
ndplas
maprotei
ns;PCC
compl
exc oncentr
ate general
lyconta
insfa
ctor
sII
,VI
I,IX,andX,as
wellasprot
einCa ndS
Cr
yopr
eci
pit
ate Cont
ainsfibr
inog
en,f
act
orVI
II,f
act
orXI
II, Co agula
ti
onfac
tordefic
ienci
esi
nvo
lvi
ng
v
WF, andfibro
nect
in fibri
nogena
ndf a
cto
rVI I
I
Bl
oodt
rans
fus
ionri
sksi
ncl
udeinf
ect
iontra
nsmi
ss
ion(l
ow),t
rans
fus
ionrea
cti
ons
,ir
onove
rlo
ad(mayl
eadto2
°
he
mochro
matosi
s)
,hypoc
alc
emia(c
itr
atei
saCa2+che
lat
or)
,andhyper
kale
mia(RBCsmaylys
einol
dblo
oduni
ts
).

Leukemia starts in the bone marrow and a lymphoma starts in lymph nodes, but they can end up in each other's locations.
Leukemi
avsl
ymphoma The pathologies are named based on where they are most likely to arise from so the major difference is the starting point.
L
euk
emi
a Lymphoidormy
elo
idneop
las
m wi
thwi
des
pre
adi
nvol
veme
nto
fbo
nema
rrow.Tumo
rce
llsa
re
usua
llyf
oundi
nperi
pher
albl
ood
.
L
ymphoma Di
scr
etet
umo
rma
ssa
ris
ingf
roml
ymp
hnode
s.Pr
ese
nta
ti
onso
fte
nbl
urd
efini
ti
ons
.
Leukemoid reaction and leukemia both have high WBC, high neutrophils, and high bands (left shift) but leukemoid reaction has high ALP and leukemia has low ALP. ALP is proof of fighting
infection. So if Leukocyte ALP is high, then it is a leukemoid reaction where the cells are trying to fight infection, but if it is low then the cells are just multiplying aimlessly.

Hodgki
nvs Hodgki
n Non-
Hodgki
n
non-
Hodgki
n Bot
hmayp
res
entwi
thc
ons
ti
tut
iona
l(“
B”)s
igns
/s
ymp toms :lo w- g rad ef ever, ni g hts we a
ts,we
ight
l
ymphoma l
oss
. "A" symptoms indicate absense of infection.

Locali
zed,si
ngl
egroupofnodes
;cont
iguous Mult
ipl
ely
mphnode
sinvo
lved;e
xtr
anodal
spre
ad(stag
eisst
rong
estpr
edi
ctorof i
nvolv
ementc
ommon;nonc
onti
guousspr
ead.
prognosi
s)
.Over
allpr
ognos
isbett
ert
hant ha
t Hodgkin has regional adenopathy and pruritis, while
non-Hodgkin has systemic adenopathy.
ofnon-Hodgki
nlymphoma.
Cha
rac
ter
ize
dbyRe
ed-
Ste
rnbe
rgc
ell
s. Majo
rit
yinv
olv
eBc
ell
s;af
ewa
reofT-
cel
l
l
inea
ge.
Bimoda
ldis
tri
buti
on–yo
ungadul
tho
oda nd Ca
noc
curi
nchi
ldr
ena
nda
dul
ts
.
>55year
s;morecommo ni
nme nexc
eptfor
nodul
ars
cle
rosi
ngtype.
As
soc
iat
edwi
thEBV. Maybeass
oci
ate
dwi t
hauto
immunedisea
ses
Biopsy largest node for diagnosis, CXR, and bone marrow biopsy.
a
ndvir
alinf
ect
ions(
eg,
HI V,EBV,HTLV).
Nodular sclerosing has best prognosis. Lymphocyte-depleted has poor prognosis.

Hodgki
nlymphoma Cont
ainsRe
ed-St
ernber
gc e
lls
:di
st
inct
ivetumo
rgi
antce
lls
;binuc
lea
teorbi
lobedwit
hthe2ha
lve
sas
A
mir
rorima
ges(“
owl e
yes”A).RSc e
llsaeCD1
r 5+andCD3 0+B-cel
lori
gin.2owley
es×15=30.
S
UBt
yPe n
ote
S

Nod
ula
rsc
ler
osi
s Mos
tcommon
Ly
mpho
cyt
eri
ch Be
stpr
ogno
sis
Mi
xedc
ell
ula
rit
y Eos
inophil
ia,s
eeni
nimmunoc
ompr
omi
sed
p
atient
s
Ly
mpho
cyt
ede
ple
ted Se
eni
nimmuno
compr
omi
sedpa
tie
nts
Prepared for students. Amir Mullick -- YouTube.com/USMLELive Best of luck.
S ectIonI I I He mat ol
ogyandonc ol ogy he matol
ogya
ndonc ol
ogy
—Pa
thology
CRUSH = B cell tumors -- SLL, Follicular Lymphoma, Diffuse Large B cell Lymphoma, Mantle Cell Lymphoma, Burkitt Lymphoma, Extranodal Marginal Zone B Cell Lymphoma or MALT.
T cell tumors -- Adult T cell lymphoma, mycosis fungoides
NK cell tumors -- Extranodal NK cell and blastic NK cell lymphomas

Non-
Hodgki
nlymphoma
t
yPe o
ccU
RSI
n g
ene
tIc
S c
omme
ntS
Neopl
asmsofma
tur
eBc
ell
s
Bur
kit
tlymphoma Adole
sce
ntso
ryo
ung t
(8;
14)
—transl
ocat
ion “
Starrysk
y”appear
a nce,sheet
sofl
ymphocytes
Burk8 14mphoma. adul
ts ofc-
myc(8)and wit
hi nt
ers
pers
ed“ ti
ngiblebody
”ma cr
ophages
hea
vy-c
hainIg(14) (
arr
owsi nA).Associat
edwi t
hEBV.
C-myc is a transcription factor which greatly upregulates cell division. Its been placed somewhere J
awle s
ionB inende micforminAfri
ca;pel
vis
it gets transcribed constantly, such as the regulation site of a heavy chain Ig site in a B cell. Now we
have way too much transcription factor. 8:22 and 2:8 are 2 similar translocations.
orabdomeninspor adi
cform.
Di
ffus
elar
geB-
cel
l Us
ual
lyol
derad
ults
, Al
ter
ati
onsi nBCL- 2, Mos tcommont y peofno n- Hodg k inl y mphoma
Aggressive and fatal. Note large cells with multi-formed nucleoli.
l
ymphoma b
ut20%inchil
dre
n BCL-6Bcl = B cell lymphoma i
na d
ults
.Treat with cyclophosphamide, doxorubicin, vincristine, or rituximab.
Fol
l
icul
arl
ymphoma Adul
ts t
(14;
18)
—tra
nsl
ocat
ion I
ndol
entcour
se;Bcl
-2inhi
bit
sapopto
sis
.
Bcl-2, as discussed on page 231, inhibits apoptosis. Upregulation ofhea
vy-
chai
nIg(14
) Pres
entswit
hpa i
nle
ss“waxi
nga ndwaning

of Bcl-2 causes complete inhibition of the cell death cascade.
Mainly composed of centrocytes (small cleaved cells) in follicles. andBCL-2(1
8) l
ymphadenopat
hy.
Mant
lec
ell
lymphoma Adul
tmals>
e >a
dul
t t
(11;
14)
—t r
ansl
ocat
ion Ve rya gg r essiv e ,pa tient st ypi c al lyp rese ntwi th
f
emal
Mantle Ce11 14mphoma.e
s ofcyc
li
nD1( 11)a
nd l a te -s
ta g edi se ase .
heavy
-chai
nIg(14)
, No centroblasts like in FL (centroblasts have prominent nucleoli and centrocytes
have cleaved nuclei). The translocation of cyclin D1 to IgG promotes progression
CD5 + of G1 to S phase, leading to proliferation. Survival of maybe 4 years.

Margi
nal
zone Adul
ts t
(1
1;1
8) Ass
oci
at
edwit
hchr
onici
nfamma t
ion(e
g,
l
ymphoma Sj
ögr
ensy
ndrome
,chro
nicga
str
it
is[MALT
l
ymphoma]
).
Pri
ma r
ycent
ral Adul
ts Mostcommo nly Considere
da nAIDS- defini
ngi ll
ness
.Vari
abl
e
nerv
oussys
tem as
soci
atedwit
hHIV/ pr es
e nt
ation:confusi
on,me mo ryl
oss
,sei
zures
.
l
ymphoma AIDS;pathog
enes
is Ma ssle
sion(s)(
ma yber ing-
e nhanci
ngin
i
nvol
v e
sEBV immuno c ompromi sedpa t
ient)onMRIC,
They are infected B cells in PCNSL.
T cells are key in AIDS related
i
nfec
tion needstobedi s
ti
ng ui
she dfr
o mt ox
opla
smo s
is
T cell lymphoma. viaCSFa nalys
isoro t
he rl
abt es
ts.
Neopl
asmsofma
tur
eTc
ell
s
Adul
tT-
cel
ll
ymphoma Ad
ult
s CausedbyHTLV Adult
spr esentwit
hcutaneousle
sio
ns;common
(
associ
at
edwit
hIV inJapan( T-cel
linTokyo)
,We s
tAfri
ca,andt
he
d
r ugabus
e) Caribbe a
n.
Lyti
cbo nele s
ions
,hyperc
alcemia.
My c
osisf
ungoides
/ Adul
ts Mycos
isfungoides
:sk i
npa t
chesandplaquesD
Sézar
ysyndrome (
cuta
neo usT-cel
llymp homa )
,char
acter
izedby
a
typi
calCD4+c ellswith“cere
bri
for
m”nuc l
ei
a
ndintraepid
erma lneoplas
ticcel
laggreg
ates
(
Pautr
iermi c
roabscess
).Ma yprog
resst
oSé zar
y
s
yndrome( T-c
elll
e ukemia)
.
A B C D
Prepared for students. Amir Mullick -- YouTube.com/USMLELive Best of luck.
Hemat ol ogyandoncol ogy hematologyandoncol
ogy—P a
thol
ogy S
ect IonI I
I

Mul
ti
plemyel
oma Mono c l
ona lpl as
mac ell(“ f
r i
ede gg” Thi nkCRAB:
appea ranc e)c ance rtha ta risesi
nt hema rr
ow Hy pe r Ca lce mi a
Ms
pik
e andp rod uc e slar
g ea mount sofIgG( 55%)o r Re na li nv olve me nt
IgA( 25%) . Bo nema r row>1 0%mo no cl
onal Ane mi a
plasmac ells.Mos tc ommo n1°tumora ri
si
ng Bo nel y ticl e s
ions /Ba c kpa in
wi t
hinbo nei npe opl e>4 0–50ye arso ld. Mul tipleMy eloma :Mo no clo na lM pr ot ein
A
lbu
minα1 α2 β γ Associatedwi th: spike
s
us c eptibili
tyt oi nfec ti
on Di sting ui s hf rom Wa ldens tröm
Prima rya my loido sis(AL) mac rogl obul inemi a Ms p ike=I g M
Punc he d-outly ti
cbo nel esi
onso nx -
rayA  hy pe rv is
c os i
tys yndr ome( eg , bl ur redv isio n,
Ms pik eons erum p roteine lect
ro phor es
is Ra yna udp he nome non );noCRABfindi ng s .
Iglig htc ha i
nsi nur ine( Be nceJo ne s
Diagnosis is with protein electrophoresis of urine (u pep) or serum (s pep)
prote i
n )
,ur i
nedi ps ticki sne g
ative( only Top left is an s pep. The M spike is due to monoclonal antibodies.
detec tsa lbumi n) Theyre IgG mostly and some IgA proteins.

Ro ule auxf orma tio nB ( RBCss t


a ck edli
ke Waldenstrom Macroglobulinemia has an M spike with IgM proteins and
hyperviscosity symptoms like mucus membrane bleeds, headaches, visual
pok erc hipsi nb loods me ar) changes, seizures, strokes, and comas and they have NO lytic bone lesions.
Nume rouspl asmac ellsC wi th“clock -face”
If macroglobulinema is seen by s pep but patient is asymptomatic, then it is
chroma tina ndi ntrac y t
o plasmicinc lusions MGUS; monoclonal gammopathy of undetermined significance.
containi ngi mmuno globul in. UWORLD= A 59yo man comes to the office due to fatigue and chronic back
Monoc lona l gammopa thyofundet er mined pain. His back pain began 3 months ago and occurs with movement. His
fatigue has been present for 3 months also, and it has been getting worse.
significanc e( MGUS )—mono cl
ona le xpansi
on Lab reveals normocytic anemia and mild hypercalcemia. Bone marrow
ofplasmac ells(bo nema r
ro w<1 0% biopsy shows tons of plasma cells. What is he at greatest risk for? Amyloidosis.
He has multiple myeloma. The characteristic appearance of plasma cells
mo noc lona lp las
mac e l
ls),asympto ma t
ic,may includes abundant basophilic cytoplasm, eccentrically placed nuclei, and
a "wagon wheel" or "clock face" distribution of nuclear chromatin. The
l
e adtomul tiplemy eloma .NoCRABfindi ngs
.perinuclear zone consists of a well-developed golgi apparatus and centrioles.
Pa t
ientswi t
hMGUSde v elopmul t
ipl e
my el
o maa tar at
eo f1 –2 %pe ryear.
A B C
A 64yo man comes to the office due to persistent
back pain, constipation, and easy fatigability for
the last several months. BP=115/75 and P=88.
He has dry mucus membranes. Lab results
show: Hb=8.6 (L), MCV=92 (N), BUN=68 (H),
Creatinine=3.8 (H), Total protein=8.9 (H), and
Albumin=4.1 (N).
Renal biopsy under light microscopy shows
atrophic tubules, many containing intensely
eosinophilic casts. What is the most likely
diagnosis? Multiple myeloma. Fatigue due to
anemia, constipation due to hypercalcemia
bone pain due to osteoclast-activating factor
by myeloma cells, elevated protein due to high
monoclonal proteins, and renal failure.

Myelodyspl
ast
ic St
emc el
ldisorder
sinvolv
inginef
fect
ive P
seudo-P e
lger-
Hueta noma
ly—neut
rophi
ls
syndromes hema t
opo i
esi
s de fec
tsincel
lma tur
ati
on wit
hbilobed(“duet”
)nucl
eiA.Typi
call
ysee
n
A
ofnonlymphoi dli
ne a
ges.Causedbydeno v
o af
terc
he motherapy
.
mut at
ionsorenvironmentalex
po s
ure(eg
,
radia
tio
n, benzene,chemother
apy).Ri
skof
tr
a ns
forma t
iontoAML.

A 67yo man is being evaluated for lower back pain and fatigue. His back pain worsens when he lies flat. His past medical history is unremarkable. Hb=8.5 (L), serum creatinine=2.2 (H), and
serum calcium=10.6 (H). After initial evaluation, he is treated with an infusion of boronic acid-containing dipeptide that has high affinity for the proteasome catalytic site. This treatment would
most likely result in what? Cell apoptosis. He has multiple myeloma. As a result of the increased protein production, plasma cells are susceptible to proteasome inhibitors like bortezomib, a
boronic acid-containing dipeptide. Proteasome inhibition favors proapoptotic proteins.
Prepared for students. Amir Mullick -- YouTube.com/USMLELive Best of luck.
S ectIonI I I He mat ol
ogyandonc ol ogy he matol
ogya
ndonc ol
ogy
—Pa
thology

L
eukemi
as Unregul
atedgro wthanddiff
erenti
ati
onofWBCsi nbo nemarrow ma r
rowfai
lur
e a nemi
a
Age is key.
( RBCs),infe
c ti
ons(ma tureWBCs )
,andhemorrhage(plate
let
s)
.Usual
lypr
e s
ent
swi t
h
 
cir
culat
ingWBCs( mal
ignantleukocyt
esi
nblood);ra
recas
espres
entwit
hnorma l
/ WBCs.
Leukemiccellinfil
tr
ati
onofli
ve r
,spl
een,l
ymphno des,ands
kin(l
eukemiacut
is
)possi
ble.
t
yPe n
ote
S
L
ymphoi
dneopl
asms
Ac ut el y mphobl astic Mos tf
requentl
yocc ursi
nc hil
dre n;l essc ommo nina d ul ts(wo rs
epr og nos is).T- cellALLc a n
l
euk emi a/lymphoma p resentasmediast
inalma ss(pre sent inga sSVC- l
ik es y ndr ome )
.As soc i
a tedwi thDowns yndr ome .
ALL -- Kids age <10.
Adolescent Little Leukemics Peri
phe r
albl
ooda ndbo nema r r
owha ve   l
ymp hob la stsA.
Age 10 Toddlers (CD10+ and TdT+)
The 12 may turn 21 and live (t(12;21) good)
TdT+( markerofpre-Tandpr e-Bc ells),CD1 0+( ma rke ro fpr e -Bc ells).--> Thymus mediastinal mass may cause dysphagia.
CRUSH = Lab Findings Mos tr
e s
ponsiv
etot hera
py. CD10 also called CALLA or neprilysin. Good prognosis. The gene for this is on Chromosome 21, so its
-- Increased WBC > 100k or less than 10k increased in Down Syndrome. CALLA is a zinc dependent metalloprotease that cleaves peptides and
-- Low Hb, low platelets. Ma yspreadtoCNSa ndtest
e s
.inactivates hormones such as glucagon, enkephalins, substance P, neurotensin, oxytocin, and bradykinin.
-- Bone marrow biopsy is diagnostic if more
than 20% blasts are seen.
t(
12;21
) b ett
erprognosis
. Spread to CNS or testes is possible because ALL cells can cross blood brain and blood testes barriers.
Chroni
clymphocyt
ic Age>60y ear
s.Mos tcommona dul
tleuk
e mia
.CD20+,CD2 3+,CD5 +B-cel
lneopl
asm.Of
ten
l
eukemia/s
mall as
ymptoma t
ic,p
rog r
ess
essl
owly;s
mudgec el
lsB i
np e
ripheralbl
oodsme a
r;aut
oimmune
l
ymphoc yt
ic hemolyt
icanemia.CLL=Cr ushedLit
tleLymphocyt
es(smudgec e
lls
).
l
ymphoma Ric
htert
ransf
ormation—CLL/ SLLtra
nsformat
ioni
ntoana ggres
si
velymphoma ,mos
tcommonly
di
ffus
elarg
eB- cel
llymphoma( DLBCL) .
Hai
ryc
ell
leukemi
a Adultma l
es.Ma t
ureB- c
elltumo r.Ce l
lshavefil
ame ntous,ha i
r-
li
kep r
ojec
tio
ns
(
fuz z
ya ppearingonLM C) .Peripherall
ympha deno pat
hyi suncommo n.
Causesma rrowfibrosi
s   dry t
apona spir
ati
on.Pat
ie ntsusuall
yp r
esentwit
hma s
si
ves
ple
nomega
ly
andpa ncytopenia.
St
ainsTRAP( tar
trat
e-r
esis
tanta ci
dpho s
phatas
e) 
⊕( tra
ppe dinaha ir
ysit
uati
on)
.TRAPsta
in
l
a r
gelyreplacedwi t
hfo wc ytome try
.As s
ocia
tedwi t
hBRAFmut at
ions.
Tr
e a
tme nt:cladri
bine,pentostat
in.
My
eloi
dneopl
asms
Acut
emyelogenous Media
nons et65years.
Aue rrodsD;my e l
oper
ox i
dase⊕ c yt
oplas
micinclusio nss eenmo stlyi n
l
eukemi
a APL(for
me rl
yM3AML) ; c ircul
ati
ngmy elobla
stsonpe r
iphera
lsme ar.
AML - Aged Men Leukemia age >60
Aged men use canes. (Auer rods). Ris
kfac
tors
:p r
iorexposuretoalk
ylati
ngc hemot
he ra
p y
,radiat
ion,myelop
ro li
fe ra tivedi sorde r
s,
AML - Auer, Myeloperoxidase, Leukemia
In APL immature cells cannot
Downs y
ndrome .APL: t
(15;1
7),r
e s
pondstoall
-tansr
r e
tinoi
ca ci
d(vit
ami nA)a nda rs e nic,whi c h
Auer rods are fused lysosomes, and spilling their
differentiate past promyelocytic stage. i
nducediff
erenti
ati
ono fpromyelocyt
es;DICisac ommo npresent
ati
on.content induces DIC, specially during chemotherapy.
Can be treated with vitamin A.
Chr oni cmye logenous Oc cur sa crosstheag espe ct
rum withpe aki ncidence45–8 5y ea
rs,me di
ana geatdiagnosi
s64y ears
.
leukemi a De finedb ythePhi la
de l
phiac hromosome( t[
9 ;
22]
,BCR- ABL)a ndmy eloi
ds t
emc e
llprol
if
era
tion.
Prese ntswi t
hdy sr
e gulatedproductiono fma tureandma turi
ngg ranul
oc y
tes(eg,neutr
ophil
s,
Occurs in patients accross the age spectrum. me tamy eloc yt
es,my elocyte
s,basophilsE)a nds p
lenome g al
y.Ma yaccele
rateandt r
ansf
ormto
Granulocytes are also across the age (some
young and some mature). AML orA LL (“bl
a stcri
sis
”).
Verylo wl euko cyt
ea lkali
nep hosphatas
e( LAP)a saresultoflowa ct
ivit
yinma l
ignantneutr
ophils
,
The only differential is leukemoid reaction,
but LAP is very low in CML and normal or vsbe nig nne utr
op hil
ia( l
eukemo idreaction)inwhi chLAPi s  dueto  leukocytecountwit
h
high in leukemoid reactions.
neut rophiliainrespons etostres
sors(eg,infecti
ons,me dicati
ons,sever
ehe mo r
rha g
e).
Respondst obc r-
abltyrosinekinaseinhibitors(eg,i
ma ti
nib, das
atini
b).
A B C D E

High lymphoblasts in ALL. Smudge cells of CLL. Hairy Cell Leukemia, fuzzy AML; Auer rods. CML; Spectra of maturing granulocytes.

UWORLD = AML, characterized by failure of immature myeloid precursors (myeloblasts) to differentiate into mature granulocytes, is dividied into 8 types (M0-M7). APML, the M3 variant of AML,
is associated with t(15;17) cytogenic translocation involving the PML gene on chromosome 15 and the retinoic acid receptor alpha (RARA) gene on chromosome 17. Fusion of these 2 genes
produces PML/RARA, a chimeric gene that codes for an abnormal retinoic acid receptor, which then inhibits myeloblast differentiation. APML is associated with DIC, which is characterized by
activation of the coagulation cascade. Lab findings in DIC include thrombocytopenia, elevated D-dimer due to fibrinolysis, and prolonged coagulation profile times and low fibrinogen due to
consumption. All-trans retinoic acid is used for treatment of APML.
Prepared for students. Amir Mullick -- YouTube.com/USMLELive Best of luck.
Hemat ol ogyandoncol ogy hematologyandoncol
ogy—P a
thol
ogy S
ect IonI I
I
The JAK2 mutation in chronic myeloproliferative disorders results in constitutive tyrosine kinase activity, and consequently in the cytokine-independent activation of signal transducers and
activators of transcription (STAT) proteins of the JAK-STAT signaling pathway. JAK2 is a cytoplasmic (non-receptor) tyrosine kinase. Patients end up having increased bone marrow
sensitivity to growth factors such as erythropoietin and thrombopoietin.

Chr
onic Themyel
opr
oli
fer
ati
vedi
sor
ders(
poly
cyt
hemiaver
a,es
sent
ialt
hromboc
ythe
mia
,my e
lofib
rosi
s,a
nd
myel
oproli
fer
ati
ve CML)aremal
ignanthe
ma t
opoi
eti
cneopl
asmswit
hvaryi
ngimpact
sonWBCsa ndmy elo
idcel
l
di
sorder
s l
ine
s.
Pol
ycyt
hemi
aver
a Pr
ima r
ypo l
ycy
themia.Disord
e rof RBCs .Ma yprese
ntasintens
ei tchi
ngaft
erhots
hower.Rar
e
butcl
ass
icsymptom i
se r
ythromelal
gia(sev
ere,burni
ngpa i
na ndre d-b
luec
olor
ati
on)dueto
epi
sodi
cb l
oodclot
sinv es
sel
softhee xtr
emiti
esA.
 
EPO (vs2°poly
cythe
mi a,whichpres
e nt
swi t
hendog eno
uso rart
ifici
all
y EPO).
Tr
eatment:phl
eboto
my ,hydroxyur
ea,ruxol
iti
nib(JAK1/2inhi
bito
r).
Es
senti
al Char
act
eriz
edbymass
iveproli
fe
rati
onofmegaka
ryocy
tesandpla
tel
ets
.Sympto
msincl
ude
t
hrombocyt
hemi
a bl
eedi
ngandthr
ombosis.Bl
oodsme ars
howsmark
edlyincr
eas
ednumbe rofp
lat
el
ets
,whi
chma
y
bel
argeorot
her
wis
eab normall
yformedB.Eryt
hromelal
giamayoccur
.
Myel
ofibr
osi
s Obl
it
era
tio
no fbonemar
rowwit
hfibros
isC duet
o fibr
obl
asta
ctiv
ity
.Oft
enass
oci
ate
dwi t
h
mas
si
vesplenomega
lya
nd“te
ardrop”RBCsD. “Bo
nema rr
owi scr
yingbec
aus
eit
’sfib
ros
eda
nd
i
sadrytap.

R
Bcs WB
cs P
lat
ele
tS P
hIl
ade
lPh
Iac
hRo
moS
ome J
AK2mU
tat
Ion
S
Pol
ycy
the
miav
era ⊝ ⊕ (100%, Somatic mutation, not inherited)

E
ssenti
al − − ⊝ ⊕(
30–
50%)
thrombocyt
hemi
a
My
elofbr
osi
s Va
ria
ble Va
ria
ble ⊝ ⊕(
30–
50%)
CML ⊕ ⊝

A B C D

Increased abnormal platelets Myelofibrosis; Bone marrow. Teardrop cells in myelofibrosis.


in essential thrombocythemia.
Relative polycythemia occurs when the number of RBC does not change but the plasma volume holding them decreases, such as in dehydration and burns.
In appropriate, the body senses low O2 levels so more EPO is made leading to an increase in RBC. The inappropriate occurs due to ectopic EPO secretion.
Pol
ycyt
hemi
a Lastly, PV occurs due to increased RBC mass which lowers EPO production and is a primary polycythemia, while the others are all secondary.
P
laS
maV
olU
me R
BcmaS
S o
2Sa
tUR
atI
on e
Pol
eVe
lS a
SSo
cIa
tIo
nS

Rel
ati
ve – – – De
hydr
ati
on,
bur
ns.
Appr
opr
iat
eabs
olut
e – Lungdis
eas
e,c
ongeni
talhe
art
di
sea
se,hi
ghal
ti
tude
.
I
nappr
opr
iat
eabs
ol e –
ut – ExogenousEPO: athl
eteabuse
(
“blooddo ping”)
.
I
nappropria
teEPO s e
cret
ion:
malignancy(eg,renalcel
l
car
cinoma ,hepatocel
lul
a r
car
cinoma )
.
Pol
ycy
the
miav
era – EPO  i
nPCVduet oneg
ati
ve
f
eed
backsuppres
si
ngre
nal
EPO p
roduct
ion.
=1
ºdi
st
urba
nce
Prepared for students. Amir Mullick -- YouTube.com/USMLELive Best of luck.

UWORLD= A 63yo woman is hospitalized because of dyspnea on exertion and generalized fatigue. She has been hospitalized 3 times during the past year for HF exacerbations. Her
other medical problems include essential HTN for 20 years, MI at age 58, and hypercholesterolemia. She has a 36 pack year smoking history in addition to a 10 year history of alcohol
abuse. Physical exam reveals tachycardia, distended neck veins, bilateral crackles on lung auscultation, a 3rd heart sound on cardiac auscultation, and pedal edema. The appropriate
therapy is initiated. On the 3rd day of hospitalization, her hematocrit level increases to 50%, up from 44% on admission. An arterial blood gas analysis shows an arterial O2 partial
pressure (PaO2) of 70. A Cr-tagged RBC infusion indicates normal RBC mass. What is the most likely cause of her polycythemia?
Relative erythrocytosis (41%, but 48% chose hypoxic erythrocytosis).

Polycythemia can be absolute (the result of a true increase in RBC mass) or relative (the result of decrease in the plasma). In a patient with erythrocytosis, the following tests can help
determine the cause:

1) Absolute vs relative= Hb and hct levels cannot accurately differentiate these conditions. Direct measurement of RBC mass is necessary. An increased total RBC mass indicates
an absolute erythrocytosis; a normal RBC mass indicates relative erythrocytosis. This patient does not have an increased RBC mass and is likely experiencing relative erythrocytosis
due to aggressive diuretic therapy for her HF exacerbation.

2) Primary vs secondary= Serum EPO levels can be used to differentiate primary from secondary erythrocytosis. Primary is associated with low EPO levels and is caused by
myeloproliferative disorders such as polycythemia vera. Secondary is characterized by increased EPO levels due to chronic hypoxia from high altitudes, smoking, or COPD; or
abnormal secretion by neoplastic or otherwise diseased tissues.

3) Hypoxic versus other causes of secondary erythrocytosis: Measurement of the arterial oxygen saturation (SaO2) is important to exclude hypoxemia as a cause of erythrocytosis.

Burkitt Lymphoma -- Burk8/14mphoma= t(8;14) c-myc;heavy Ig -- Starry Sky, EBV, Jaw lesions in Africa, pelvic or abdominal in sporadic form.
Diffuse DLBCL -- Alterations in Bcl-2 Bcl-6= Most common non-Hodgkin, aggressive, fatal.
Follicular Lymphoma -- t(14;18) heavy Ig;Bcl-2 = Indolent, waxing and waning, painless.
Mantle cell lymphoma -- ce11 14mphoma= t(11;14) cyclin D1; heavy Ig = CD5+, very aggressive.
PCNSL -- HIV/AIDS, Atypical infected B cells (reactive T cells in EBV).

Adult T-cell Lymphoma -- HTLV and IV drug abuse. AIDS. Ring-enhancing lesions on MRI.
Mycosis fungoides/Sezary syndrome -- Skin patches, cerebriform nuclei CD4+ cells, neoplastic cell aggreggates (Pautrier microabscess).

LYMPHOID NEOPLASMS ALL -- Kids under 10, CD10, TdT, t(12;21), CNS/Testes spread, high lymphoblasts, mediastinal mass.
CLL -- Adults over 60, most common adult leukemia, CD20, CD23, CD5, smudge cells, autoimmune hemolytic anemia.
Hairy Cell Leukemia -- Adult males, no peripheral lymphadenopathy, dry tap, TRAP+, cytometry more common, cladribine/pentostatin.

MYELOID NEOPLASMS -- AML -- Age 65, Auer rods of myeloperoxidase, high myelooblasts, DIC, t(15;17) responds to vitamin A.
CML -- t(9;22) BCR-ABL, across age spectrum, dysregulated production of maturing granulocytes, low LAP, responds to bcr-abl tyrosine kinase inhibitor imatinib, dasatinib.
Prepared for students. Amir Mullick -- YouTube.com/USMLELive Best of luck.
S ectIonI I I He mat ol
ogyandonc ol ogy he matol
ogya
ndonc ol
ogy
—Pa
thology

t(12;21) Acute Lymphoblastic Leukemia/ Lymphoma CD10/TdT/Thymus mass/Down Syndrome/Good prognosis/CNS & testes spread/Elevated Lymphoblasts.

Chr
omos
omal
trans
loc
ati
ons
t
Ran
Slo
cat
Ion a
SSo
cIa
tedd
ISo
Rde
R n
ote
S
t
(8;
14) Bur
kit
t(Bur
k-8
)ly
mpho
ma(
c-
myca
cti
vat
ion) TheIghe avycha i
ngenesonc hr
omo some 
14
t
(1
1;1
4) Ma
ntl
ece
lll
ymp
homa(
cyc
li
nD1a
cti
vat
ion) ar
ec onsti
tuti
velyex
pres
sed. Wheno t
her
ge
ne s(e
g ,c-
my candBCL- 2)ar
etrans
loca
ted
t
(1
1;1
8) Ma
rgi
nalz
onel
ymp
homa
nexttothisheavychai
ng eneregi
on,the
ya r
e
t
(1
4;1
8) Fo
lli
cul
arl
ympho
ma(
BCL-
2ac
tiv
ati
on) ov
erexpressed
.
t
(1
5;1
7) APL(M3ty
peofAML;r
espondst
oal
l-
tr
ans
r
eti
noi
cac
id)
t
(9;
22)(Phil
ade
lphi
a CML( BCR-ABLhybr
id),ALL(l
essc
ommon,
chromosome) p
oorprognos
ti
cfa
cto
r);Phil
ade
lphi
aCr
eaML
c
hees
e .

Langerhanscel
l Coll
ectiv
eg ro
upo fpr
olife
rati
vedisordersof A B
his
tioc
ytosis Langerhansc e
lls
.Presentsinac hil
da slyti
c
bonelesi
onsA a nds ki
nr ashora srec
ur re
nt
oti
ti
sme diawithama ssinvol
vingthema st
oid
bone.Ce l
lsarefuncti
o nall
yimma t
urea nd
donote f
fect
ivelyst
imul at
eprima ryTc ell
s
vi
aa nti
genpr es
entat
ion. Cell
se xpr
essS-100
(
me sodermalo r
igin
)a ndCD1 a
.Bi rbeck
gra
nules(“tennisra
cke ts
”orrod shapedo n
EM)a rechar a
cteri
st
icB.

T
umorl
ysi
ssyndr
ome
Mu
scl
e Onc ol
o g
ice me r
g encytri
ggeredbyma ss
ive
we
akn
ess tumo rcel
llysis,mo stoft
eninl y
mphoma s/
+
K
leukemias.Re l
e a
s eofK+   hyperkalemia,
A
rrh
ythmi
as
,
E
CGc h
ange
s
relea
seofPO43–   hyperphosphat
e mia,
hypoc a
lcemi aduet oCa 2+sequest
ration
2
Ca+

S
eiz
ure
s, byPO43–.  nuc l
e i
ca c
idbreakdown
T
umorcel
l
t
et
any  hyper
ur i
cemi a   acutekidneyinjury
.
Cal
ci
um
l
ys
is P
O43

phos
pha
te Preventi
ona ndt reatmentincludeagg r
essi
ve
cr
yst
al
s A
cut
eki
dne
y hydrati
on, al
lopur i
no l
,ra
sburicas
e.
i
nj
ur
y
u
ri
cac
id Ur
icaci
d The principal site of uric acid precipitation would be where?
cr
yst
al
s Collecting ducts due to low urine pH. Uric acid (pKa = 5.4) is soluble
at physiologic pH, but precipitates in an acidic environment. The lowest
pH along the nephron is found in the distal tubules and CD.

Note patient has weight loss, fatigue, bruising, and nosebleed. So think leukemia.

Patient is 50, so cancel ALL and CLL. AML and CML are likely.

His first metatarsophalangeal pain hints he has gout so uric acid is building up.

Full spectrum of myeloid cells is key. AML has large accumulation of parts of lineage cells, but CML
has abundance of all the lineage cells. The philadelphia chromosome is involved in CML and this
guy has a translocation so diagnosis is most likely CML.

Translocation must be the philadelphia chromosome and treatment imatinib, a tyrosine kinase inhibitor.

Xanthine oxidase inhibitor wouldnt treat the persons condition, which is his cancer not his gout only.
Prepared for students. Amir Mullick -- YouTube.com/USMLELive Best of luck.
Hema tol ogyandonc ology hema tol
ogyandonc olog
y—PhaR mac
ology S
ect IonI I
I

Hemophagocyti
c Syst
emicover
act
iva
tionofmacrophagesandcyt
otox
icTcell
s feve
r,pancy
topeni
a,
l
ymphohist
iocyt
osi
s hepat
ospl
enomegaly
.Canbei nheri
tedor2°tost
rongi
mmunolog
icacti
vat
ion(eg
,af
terEBV
A inf
ect
ion,mali
gnancy)
.Bonema rrowbiops
yshowsmacro
phag
esphagocyt
osi
ngma r
row
ele
me nt
sA.   s
erum fe
rri
ti
nl ev
els.

he
mat
ologya
ndonc
ology
—PhaR
mac
olog
y

Hepar
in
me
cha
nIS
m Ac
tiv
ate
sant
it
hrombi
n,whi
ch 
act
ionofI
Ia(
thr
ombi
n)a
ndf
act
orXa
.Shor
tha
lf-
li
fe.
c
lI
nI
calU
Se I
mme di
atea
nti
coa
g ul
ati
onf
orpul
monarye
mboli
sm(PE)
,acut
ecor
onarys
yndrome,MI,d
eep
ve
noust
hrombos
is(DVT).
Useddur
ingpr
egna
ncy(
doesnotc
ros
spla
cent
a).Fol
lowPTT.
a
dVe
RSee
FFe
ctS Bl
eedi
ng ,
thrombocy
topeni
a(HIT)
,os
teoporo
sis
,drug
-dr
ugint
era
c t
ions
.Fo
rrapi
dr e
ver
sal
(
anti
dote
),usepr
ota
mi nes
ulf
ate(
pos
it
ivel
ycharge
dmolecul
ethatbi
ndsnega
tiv
elycha
rge
d
hep
arin)
.
n
ote
S Low-molecul
ar-
weighthepar
ins(e
g ,
enoxapari
n,dal
tepar
in)actpr
edominant
lyonfact
orXa.
Fondapar
inuxactsonlyonfac
torXa.Havebe t
terbi
oavai
lab
ili
tyand2–4×long
erhalfli
fet
han
unfr
acti
onate
dhe pari
n;canbea dmini
st
e r
edsubcut
a ne
ouslyandwit
houtla
borat
orymonit
ori
ng.
Heparin stops 2a and 10a.
LMWHs and fondaparinux stop 10a.
Noteasi
lyrev
ersi
ble.
Hepari
n-i
nducedthr
omboc ytopenia(
HIT)—de v
elopmento fI
gGa nti
bodie
saga
insthepa
rin-
boundpl
ate
letf
act
or4(PF4).Ant
ibody
-hepa
rin-
PF4c omplexact
iva
tespla
tel
ets thrombosi
sand
t
hrombocyt
openi
a.Hig
hestris
kwithunfr
acti
onat
edhe pa
rin.

Can take Dabigatran in place of warfarin to prevent stroke or a-fib. Does not require periodic monitoring like warfarin.

Di
rectthr
ombi
n Bi
val
ir
udin(
rel
ate
dtohi
rudi
n,t
hea
nti
coa
gul
antus
edbyl
eec
hes
),Ar
gat
rob
an,Da
big
atr
an(
onl
y
i
nhibi
tors o
ralag
enti
nclass
).
me
cha
nIS
m Di
rec
tlyi
nhi
bit
sac
tiv
ityoff
reea
ndc
lot
-a
ssoc
iat
edt
hro
mbi
n.
c
lI
nI
calU
Se Ve
noust
hro
mbo embol
is
m,atr
ialfibr
il
lat
ion.
Canbeus
edi
nHI
T,whe
nhe
par
i sBAD f
ni ort
he
p
ati
ent
.Doesnotr
equi
rel
abmoni t
ori
ng.
a
dVe
RSee
FFe
ctS Bl
eedi
ng;c
a nr
ever
seda
bigat
ranwi
thi
dar
uc i
zuma
b.Co
nsi
derPCCa
nd/
ora
nti
fib
rino
lyt
ics(
eg,
t
rane
xamicac
id)i
fnorev
ersa
lage
nta
vai
labl
e.

Heparin has the greatest efficacy on thrombin (2a) while


LMWH has the greatest efficacy on factor 10a.
Heparin works on both via antithrombin.

The anitbodies bind Fc receptors on platelets and some of them are


removed and destroyed by the spleen, leading to thrombocytopenia,
while other complexes remain in the bloodstream in an over-activated
state leading to thrombi and cause coagulation. So you get both
thrombocytopenic and hyper-coagulable state. If a patient recently
starting on heparin gets a low platelet count, stop heparin asap and
use a non-heparin derivative quick-acting anticoagulant, such as direct
thrombin inhibitors.
Prepared for students. Amir Mullick -- YouTube.com/USMLELive Best of luck.
S ectIonI I I He mat ol
ogyandonc ol ogy he matol
ogya
ndonc ol
ogy
—PhaRmac ology

War
far
in
me
cha
nIS
m I
nhi bit
sepoxider e
d uct
ase,whichint
erfer
es TheEX- Pr e si
de nTwe ntt owa r(farin).
wi hγ
t -ca
rboxylat
ionofv i
taminK–de pendent
UWORLD= A 3wk old boy is brought to the ED due to lethargy for a day. The patient
clotti
ngfactorsII
,VI I
,IX, X,andprote
insC, was breastfeeding well until this morning, when he became increasingly difficult to
S.Me ta
boli
s ma f
fecte
db ypo l
ymo r
phisms arouse. He was born full-term to a 35yo mother who had spontaneous vaginal
delivery at home. She took prenatal vitamins throughout pregnancy and received
int hegenef o
rv i
taminKe poxi
dereductase routine prenatal care. However, the boy did not receive any vaccinations or
medications after birth due to parental preference for a "natural" newborn period.
comp l
ex(VKORC1 )
.Inlaborat
oryass
ay,has Family history is unremarkable. Head circumference is at the 99th % and weight &
effectonEXt rins
icpa t
hwa yand PT.Lo ng height are at 25th %. Physical exam shows a large, bulging fontanel. The eyes are
driven down and the patient cannot track upward. No scalp swelling is present.
ha l
f-l
if
e. Intracranial hemorrhage is confirmed on head CT scan. What is the cause of this
infant's condition? Impaired clotting factor carboxylation. Vitamin K is essential for
c
lI
nI
calU
Se Chronicanti
coagulati
on( e
g,venous gamma carboxylation of clotting factors 2, 7, 9, and 10 so they can bind calcium and
attract negative phospholipids on platelets and endothelial cells.
thr
ombo embolism prophyl
axi
s,andp r
event
ion
ofst
rokeinatr
ialfibri
ll
ati
on).
No tusedin Neonatal vitamin K deficiency presents with intracranial, GI, cutaneous, umbilical,
and surgical site bleeding. Prevent with intramuscular vitamin K at birth.
preg
na ntwome n(be ca
usewarfa
rin,unli
ke
hepari
n, c
ros
sespla c
enta)
.Foll
owPT/ INR.
a
dVe
RSee
FFe
ctS Ble
e ding ,t
erat
og enic,skin/t
issuene c r
osisA, Forrevers
alo fwarf
a ri
n, givevit
ami nK.
A
drug -
dr uginte
r a
c ti
ons . Forrapidreversa
l,givef r
eshf r
oze npla
sma
Ini
ti
a lriskofhype rcoagulati
o n: pr
ote i
nC (FFP)orPCC.
hasas hort
erha l
f-li
fetha nfac to
rsI IandX. He pa
rin“ b
r i
dging”
: he pari
nf r
eq uent
lyused
Existingpr ot
einCde p l
e t
esbe foree xi
sti
ng whe nstar
tingwa rfari
n. He par
in’sact
ivat
ionof
fa
c t
o rsIIandXde pl
ete,andbe forewa rf
arin anti
thromb inenab lesanticoagulat
iondur i
ng
canr ed ucefacto
r sIIandXp roduction ini
tial
,transie
nthy pe rc
o agul
ablestat
ec aused
 
hy pe rc
oagulation.Sk in/ti
ssuene cr
osis bywa rfa
rin.Init
ialhe parintherapyreduces
withinfirstfe
wda yso flarg
ed osesbe l
ievedto ris
ko frec
ur re
ntve noust hromboe mbo l
is
m
bed uet osma llve s
selmi crothrombos is. ands ki
n/tis
suene crosis
.
Me t
abolizedbyc yt
oc hr omeP- 450.

Hepar
invswar
far
in
Hepar
in Wa
rfa
rin
R
oUt
eoFa
dmI
nI
St
Rat
Ion Pa
rent
era
l(I
V,SC) Or
al
S
It
eoFa
ctI
on Bl
ood Li
ver
o
nSe
toFa
ctI
on Ra
pid(
sec
onds
) Sl
ow,l
imit
edb
yha
lf
-l
iv
eso
fno
rma
lcl
ott
ing
f
act
ors
me
cha
nIS
moFa
ctI
on Ac
tiv
atesa
nti
thr
ombi
n,whi
ch t
hea
cti
onof I
mpa i
rssy
nthes
isofvi
taminK–de
pende
nt
I
Ia(t
hrombi
n)andf
act
orXa cl
ott
ingfac
tor
sI I
,VI
I,IX,a
ndX,andant
i-
cl
ott
ingprot
einsCandS
d
URa
tIo
noFa
ctI
on Ho
urs Da
ys
a
gen
tSF
oRR
eVe
RSa
l Pr
ota
mines
ulf
ate Vi
tami
nK,
FFP
,PCC
mo
nIt
oRI
ng PTT(
int
ri
nsi
cpa
thwa
y) PT/
INR(
ext
ri
nsi
cpa
thwa
y)
c
RoS
SeSP
lac
ent
a No Y
es(
te
rat
oge
nic
) Normal INR is 1.1 or below. An INR of 2 to 3 is an
effective therapeutic range for people taking warfarin.
A higher INR indicates that your blood clots slowly.

UWORLD= A 24yo woman comes to the ED due to left leg swelling and pain for the past 2 days. The patient is at 14 weeks gestation and has had an uneventful pregnancy to date. She has
no chest pain or shortness of breath. The patient's past medical history is otherwise unremarkable. She is a lifetime nonsmoker. Physical examination shows 1+ edema of the left lower
extremity to the knee, associated with erythema. The patient's left calf diameter measures 3cm greater than the right. Duplex ultrasound examination is consistent with left popliteal and femoral
vein thrombosis. Her serum creatinine is 0.7mg/dL. Which of the following is the best treatment option for this patient?
Apixaban, Clopidogrel, Dabigatran, Enoxaparin, Low dose aspirin, Tissue plasminogen activator, warfarin.

Enoxaparin. Pregnancy increases the risk of venous thromboembolic disease due to increased venous stasis, endothelial injury, and hypercoagulability. As the uterus grows it compresses
the IVC and internal iliac veins, reducing flow from the lower extremities. Hypercoagulability is due to increased factors 1, 2, 7, 8, 9, 10, and a drop in protein S. LMWHs currently provide the
best balance of risks and benefits to the mother and child both. In addition, LMWHs also have a fairly simple mode of administration, and good bioavailability, as well as a short elimination time.
Prepared for students. Amir Mullick -- YouTube.com/USMLELive Best of luck.
Hema tol ogyandonc ology hema tol
ogyandonc olog
y—PhaR mac
ology S
ect IonI I
I

Di
rectfac
torXa
i
nhibi
tors Ap
iXa
ban,
riv
aroXa
ban.
me
cha
nIS
m Bi
ndt
oanddi
rec
tlyi
nhi
bitf
act
orXa
.
c
lI
nI
calU
Se Tr
eatmentandpr
ophyl
axi
sfo
rDVTa ndPE;st
rok
eprophyl
axi
sinpa
tie
ntswi
tha
tri
alfibr
il
lat
ion.
Oralag
entsdono
tusual
lyr
equi
rec
oagul
ati
onmo ni
tor
ing
.
a
dVe
RSee
FFe
ctS Bl
eed
ing
.Note
asi
lyr
eve
rsi
ble
.

Thr
ombol
yti
cs Al
tep
las
e(t
PA)
,re
tep
las
e(r
PA)
,st
rept
oki
nas
e,t
ene
cte
pla
se(
TNK-
tPA)
.
me
cha
nIS
m Di
rect
lyori
ndi
rec
tlya
idco
nvers
ionofpl
asminog
ent opl asmi n, whi c hc lea ve sthr ombi na ndfib ri
n
c
lot
s. PT, PTT,nochang
einp l
at
ele
tcount.
Bleeding time will not be affected since platelet count is unchanged.

c
lI
nI
calU
Se Ea
rlyMI
,ea
rlyi
sche
mics
tr
oke
,di
rec
tthr
ombo
lys
isofs
eve
rePE.
a
dVe
RSee
FFe
ctS Bl
eed
ing. Contr
aindica
tedinpati
ent
swithacti
vebl
e e
ding,his
toryofi
ntr
acr
ani a
lblee
ding,
r
ece
nts urg
ery,knownb l
eedi
ngdiat
he s
es,ors
ever
ehy per
tensi
on.Nonspec
ificrev
ers
alwith
Main danger is lysing clots the body actually a
nti
fibr
inoly
tic
s(eg,aminocapr
oicaci
d,tr
anexamicacid)
,pla
tel
ettra
nsf
usi
ons ,a
ndfactor
needs so be careful with patients who have
history of bleeding or had a surgery. c
orr
e c
tions(e
g,c r
yoprec
ipi
ta
te,FFP,PCC).

ADPr
ecept
ori
nhi
bit
ors Cl
opi
dog
rel
,pr
asug
rel
,ti
cag
rel
or(
rev
ers
ibl
e)
,ti
cl
opi
dine
.
me
cha
nIS
m I
nhibitp
lat
ele
taggr
egat
ionbyi
rrev
ers
ibl
ybloc
kingADP(
P2Y12)r
ece
ptor
.Pr
eve
nte
xpr
ess
ionof
gl
ycoprot
ei
nsII
b/I
IIaonpl
ate
lets
urf
ace.
c
lI
nI
calU
Se Ac
utec
orona
rys
yndr
ome
;co
rona
rys
tent
ing
.inc
ide
nceo
rre
cur
renc
eoft
hro
mbot
ics
tr
oke
.
a
dVe
RSee
FFe
ctS Ne
utr
ope
nia(
ti
cl
opi
dine
).TTPma
ybes
een.

Anti
platelet
phosphodi es
ter
ase
i
nhibit
or s Ci
los
taz
ol,
dip
yri
damo
le. Usually used in combination with aspirin. Dipyridamole is also used in the coronary stress test.
me
cha
nIS
m c
AMPi
npl
ate
let
s,r
esul
ti
ngi
ninhi
bit
ionofp
lat
el
eta
ggr
ega
tio
n;v
asodi
la
tor
s.
c
lI
nI
calU
Se I
nte
rmitt
entcl
audic
ati
on,c
oro
nar
yvasodi
la
ti
on(di
pyr
ida
mol
eus
edf
orc
ardi
acs
tr
esst
est
ing
),
pr
eve
ntionofs
trok
eorTIAs(
combi
nedwithas
pir
in)
.
a
dVe
RSee
FFe
ctS Na
use
a,he
ada
che
,fa
cia
lfus
hing
,hy
pot
ens
ion,
abdo
mina
lpa
in.
The net effect is a decrease in claudication and an increase in pain-free walking distances in peripheral artery disease.
Pulsatile cAMP activates platelets, but high levels inhibit platelet aggregation.

Gl
ycoprot
einI
Ib/
II
Ia
i
nhibi
tors Abc
ixi
mab,e
pti
fiba
tide
,ti
rofiba
n.
me
cha
nIS
m Bi
ndt
othegl
ycopr
ote
inrec
ept
orII
b/I
IIao
na ct
iva
tedpl
at
ele
ts,p
rev
ent
inga
ggr
ega
ti
on.
Abc
ixi
mab
i
smadef
rom monoc
lona
lant
ibod
yFabfragments
.
c
lI
nI
calU
Se Uns
tab
lea
ngi
na,pe
rcut
ane
ousc
orona
ryi
nte
rve
nti
on.
a
dVe
RSee
FFe
ctS Bl
eed
ing
,thr
ombo
cyt
ope
nia
.
Prepared for students. Amir Mullick -- YouTube.com/USMLELive Best of luck.
S ectIonI I I He mat ol
ogyandonc ol ogy he matol
ogya
ndonc ol
ogy
—PhaRmac ology

Canc
erdr
ugs
—cel
lcyc
le

B
leo
myc
in Mi
crot
ubulei
nhi
bi
tor
s
P
acli
ta
xel
V
inbl
ast
in
e
V
incr
is
ti
ne
E
ri
b u
li
n
G2 Mi M
t
os
Do
ubl
echec
k i
s
re
pai
r
is
nes
oki
Cyt
T
opois
ome r
asei
nhi
bi
tor
s
Et
oposi
de

I
NT
T
enip
osi
de RP
E
I
ri
not
ecan HASE Dupl
ica
te
T
opotec
an DNA c
ell
ul
arc
onte
nt
s
ynt
hes
is Cel
lcycl
e–i
ndependentdru
gs
G1 Pl
at
inu
ma gent
s(eg
,ci
s pl
at
in
)
A
nti
metabol
it
es GO Al
kyl
at
ingagen
ts:
Az
athi
opri
ne S R
est
i
n g Bus
ulf
an
Cl
adr
ibi
ne Cyc
lophos
phamide
Cy
tar
abine I
fos
famide
5-
fluoro
u r
aci
l Rb,p5
3mo du
lat
e Ni
tr
osoure
as(e
g,car
mu st
in
e)
Hydr
oxyur
ea G1r
est
ri
ct
io
np oi
nt
Met
hotr
exat
e
6-
mercapt
opur
in
e

Canc
erdr
ugs
—tar
get
s
Nu
cle
oti
des
ynt
hes
is DNA R
NA P
rot
ei
n Ce
ll
ul
ard
ivi
si
on

MTX
,5-FU: V
inc
aalk
alo
ids
:
A
lkyl
at
in
ga ge
nts
,pl
at
in
uma
gen
ts:
↓thy
mi d
ines
ynt
hes
is i
nhi
bi
tmicr
otu
bul
efo
rma
ti
on
cr
oss
-l
inkDNA

6
-MP: P
acl
it
axel
:
B
leomy
cin
:
↓d
en o
vop
uri
nes
ynt
hes
is i
nhi
bi
tsmic
rot
ubu
led
is
ass
emb
ly
DNAs
tr
andb
rea
kag
e

Hy
dro
xyur
ea: Dac
ti
nomyci
n,d
oxoru
bic
in:
i
nhi
bi
tsr
ib
onuc
leo
ti
de DNAi
nte
rca
lat
ors
r
edu
ctas
e
E
topo
side
/te
nip
osi
de:
i
nhi
bi
tsto
pois
omer
aseI
I

I
r
inot
ecan
/to
potec
an:
i
nhi
bi
tst
opois
omera
seI
Prepared for students. Amir Mullick -- YouTube.com/USMLELive Best of luck.
Hema tol ogyandonc ology hema tol
ogyandonc olog
y—PhaR mac
ology S
ect IonI I
I

Ant
itumorant
ibi
oti
cs
d
RUg me
cha
nIS
m c
lIn
Ica
lUS
e a
dVe
RSee
FFe
ctS
Bl
eomyc
in I
nducesf
reera
dic
alf
ormat
ion Te
sti
cul
arc
ance
r,Ho
dgk
in Pul
mona r
yfibr
osis,s
kin
 b
reaksi
nDNAs t
rands
. l
ymphoma. hy
perpigmentat
ion.Mini
mal
myel
osuppr
e s
si
on.
Dact
inomyc
in I
nte
rcal
atesi
ntoDNA, Wil
mst umor,Ewingsar
coma, My elo suppr ess io n.
(
act
inomyc
inD) pr
eve
ntingRNAsynt
hes
is
. r
habdomy o
sarc
oma .Use
dfor Since this is used for childhood tumors use
c
hil
dho odtumors. the pneumonic "Children act out."

Doxorubici
n, Genera
tefr
eera
dicals
. So
lidt
umor
s,l
euk
emi
as, Car
diotox
ici
ty(dil
ated
daunorubi
cin I
nter
cala
teinDNA b r
eaksi
n lymphoma
s. ca
rdiomyopathy)
,
DNA r
epl
ica
tion. myel
o s
uppres
s i
on,alo
pec i
a.
I
nter
fer
eswitht
opoisomer
aseI
I Dexr
azoxane(ironchela
ting
enz
yme . ag
ent)usedtoprevent
ca
rdiot
oxic
ity
.
Prepared for students. Amir Mullick -- YouTube.com/USMLELive Best of luck.
S ectIonI I I He mat ol
ogyandonc ol ogy he matol
ogya
ndonc ol
ogy
—PhaRmac ology

Ant
imet
abol
it
es S Phase

d
RUg me
cha
nIma
S c
lI
nI
calU
Se a
dVe
RSee
FFe
ctS
Azat
hiopr
ine, Puri
ne( t
hio
l)ana
logs Preventi
ngo rganrej
ecti
on, Myel
os uppres
si
on;GI,l
iver
6-
mercaptopur
ine   de 
novopuri
nesynt
hesis
. rheuma t
oida rt
hrit
is
,IBD, t
oxic
ity.
Acti
vate
dbyHGPRT. SLE; usedt oweanpa t
ients Az
athiopri
nea nd6-MPa r
e
Azathi
opri
neismetabol
ize
d offste
roidsinchronicdiseas
e me taboli
zedbyxanthi
ne
int
o6-MP . andt otr
e atst
eroi
d-r
efr
ac t
ory o
xidase;thusbot
hha v
e  ri
sk
Toxic to patients treating gout, chronicd i
sease. o
ftoxicit
ywi t
hall
opuri
nolor
so give 6-Thioguanine, which is not
metabolized by xanthine oxidase. f
ebuxostat.
Cl
adr
ibi
ne Puri
nea nal
og  multi
ple Ha
iryc
elll
euk
emi
a. Myel
osuppre
ssi
on,
mec ha
nisms(eg
,inhi
bit
ion Pentostatin is the other drug for hairy cell, and nep
h r
otox
ici
ty,a
nd
Give 0.1mg/kg/d by continuous it is an adenosine analog that inhibits adenosine n
infusion via IV for 7 days. ofDNApol ymeras
e,DNA deaminase. It is given IV once every 2 weeks for
e
u r
otox
ici
ty.
st
randb r
eaks
). 3-6 months and can also help with graft-vs-host.

Cytar
abine Py
r i
midineanal
og DNA Le
uke
mia
s(AML)
,ly
mpho
mas
. Myel
osuppr
essi
onwith
(
arabinof
uranos
yl c
ha i
nt er
minat
ion.Athighe
r megal
obla
sti
c a
nemia.
cyti
dine) c
onc entr
ati
ons
,inhib
itsDNA CYTarabi
necause
s
poly
me ras
e. pnCYTo
a penia
.
5-
fuor
our
aci
l Pyri
mi di neana l
ogbioac
tiv
ate
d Colo nc a nc e r, pa nc re at
ic Myel
osuppr
ess
ion,palmar
-
TS is a key enzyme in the creation to5-FdUMP ,
which canc e r,a c tini ck e ratosis, ba s al pl ant
areryt
hrody
s e
sthe
sia
of thymine, a building block of DNA.
5-FU competes with the natural covale ntl
yc omplex
eswith cellc a rc i
noma( topi cal). (
hand-f
oots
yndrome).
substrate to bind the TS active site, thymi dy l
atesynt
haseand Ef
fe ctse nha nc edwi tht he
arresting DNA synthesis. But 5-FU
alone can be displaced, enabling fol
ica c i
d.Ca peci
ta
bineisa a
ddi tio nofl e uc ov or i
n.
DNA synthesis in cancer cells.
prodr ug.
Leucovorin is readily converted to Thisc ompl exinhibi
ts
5-MTHF, which stabilizes the bond Hand-foot syndrome is a blistering
between 5-FU and TS, enhancing thymi dy l
atesynt
hase ulceration of skin with normal activities.
its inhibition of the enzyme.
  d TMP DNA
synthe sis
.
Met
hot
rexa
te Foli
ca c
idanalo
gt ha
t Canc e
r s:leuk emias My elosupp res
si
o n,whi chis
compe t
it
ive
lyinhib
its (
ALL) ,lymp homa s
, reversiblewithl eucovori
n
dihydr
ofol
atereduc
tase chorioc arcinoma ,sar
comas. “rescue .

  dTMP    DNA Non-ne o plasti
c:ectopi
c He patotoxici
ty.
synt
he s
is
. pregna nc y,me di
c a
l Muc os it
is(eg,mout hul ce
rs)
.
abortion( wi t
hmi sopros
tol
), Pul mo naryfibr
o sis.
r
he uma toida r
thrit
is,
psori
asi
s, Folatede fic
iency, which
I
BD,v a sculiti
s. ma ybet erat
oge nic(neura
l
tubede f
e ct
s)witho ut
suppl eme ntat
ion.
Ne phrotox i
cit
y.
a
Alla
reS-
pha
ses
pec
ifice
xce
ptc
ladr
ibi
ne,whi
chi
sce
llc
ycl
enons
pec
ific
.

Allopurinol inhibits Xanthine Oxidase to decrease uric acid production. But XO also
metabolizes drugs that mimic purines, like the drug 6-Mercaptopurine, which treats
leukemias, IBD, and RA. Giving this patient allopurinol will lead to a build-up of 6-MP
so lower the 6-MP dose there. 6-thioguanine is also inactivated by XO.
Both 6MP and 6TG are affected by HGPRT and TPMT.
Page 59 - biochemistry.
Prepared for students. Amir Mullick -- YouTube.com/USMLELive Best of luck.
Hema tol ogyandonc ology hema tol
ogyandonc olog
y—PhaR mac
ology S
ect IonI I
I

Al
kyl
ati
ngagent
s
d
RUg me
cha
nIS
m c
lIn
Ica
lUS
e a
dVe
RSee
FFe
ctS
Bus
ulf
an Cr
oss
-l
ink
sDNA. Use
dtoabla
tepati
ent’
sbo
ne Sev e
remy el
osuppress
ion(i
n
marr
owbefor
ebo nemarr
ow almos tal
lcases
),pulmonary
t
rans
pla
ntat
ion. fibros
is,
hyperpigmentat
ion.
Cycl
ophospha
mide, Cro
ss-l
inkDNAa tguani
ne. Sol
idtumors,l
e uke
mia, My el
osuppr e s
sion;SIADH;
i
fosf
amide Requir
ebioa
cti
vat
ionbyli
ver
. l y
mpho mas,rheumati
c Fanco nis yndrome
Anitr
ogenmustar
d. dis
ease(
eg,SLE, (i
fosf
ami de);hemor rhagi
c
gra
nulomatosi
swith cyst
iti
sa ndbl a
dd ercancer,
poly
angi
it
is). preventedwi t
hme sna
(
sulfhy drylg r
oupofme s
na
bindstox icme tabolit
es)and
adequa tehy drati
on.
Ni
tros
our eas Re
quir
ebioa
c t
ivat
ion. Br
aintumors(
inc
luding CNStox
ici
ty(convul
si
ons
,
(
eg,car
mus ti
ne, Cr
ossbl
ood-
brainb ar
ri
er g
liob
las
tomamultif
orme)
. d
izz
ines
s,at
axia)
.
l
omus t
ine) CNS.Cross
-li
nkDNA.
Pr
ocar
baz
ine Cel
lcycl
epha se–
nons
peci
fic Hodg
kinly
mpho
ma,b
rai
n Bonema r
rows uppr
essi
on,
al
kyl
ati
nga gent
,mec
hanism t
umors
. p
ul mo narytoxi
cit
y,l
euke
mia
,
noty
etdefine
d. d
i s
ulfir
am- l
ikereac
tio
n.

Mi
crot
ubul
einhi
bit
ors M Phase
d
RUg me
cha
nIS
m c
lIn
Ica
lUS
e a
dVe
RSee
FFe
ctS
Pacl
i
taxel
,ot
her Hypers
tabil
izepoly
me r
ized Ov
ari
ana
ndb
rea
stc
arc
inoma
s. Myel
osuppre
ssi
on,neuropa
thy
,
t
axanes micr
otubulesinM phaseso hy
persens
it
ivi
ty
.
t
hatmitoti
cs p
indl
ecanno t Tax
esstabi
li
zesoci
ety.
br
eakd own(anaphas
ec annot
occ
ur).
Vi
ncr
ist
ine,
vinbl
ast
ine Vincaa l
kaloi
dsthatb i
nd Sol
idt
umo r
s,l
euk
emias
, Vincri
sti
ne :neurotox
icit
y
β-t
ub uli
na ndinhibit Hodgki
nandnon-Ho
dgki
n (
a r
efexia,pe r
iphera
lne ur
it
is
),
i
tspolyme ri
zati
oni nt
o l
ymphomas. const
ipation(incl
uding
microtubules  prevent paral
yti
ci leus
).Crispsthe
mitoti
cs p
indlefor
ma t
ion nerves
.
(M-pha s
ea rr
est
). Vinblas
tine :
bonema rr
ow
suppres
sion.Bl as
tsthebone
ma rr
ow.

Ci
spl
ati
n,c
arbopl
ati
n,oxal
ipl
ati
n
me
cha
nIS
m Cr
oss
-l
inkDNA.
c
lI
nI
calU
Se Te
sti
cul
ar,
bla
dde
r,o
var
y,GI
,andl
ungc
arc
ino
mas
.
a
dVe
RSee
FFe
ctS Nephro
toxi
cit
y(i
ncl
udi
ngFanconisyndro
me )
,pe
ripher
alneurop
athy,ot
otox
ici
ty
.Prev
ent
nephr
oto
xici
tywit
hamif
ost
ine(f
reeradi
cals
cav
eng e
r)andchlor
ide(s
ali
ne)di
ures
is
.
Prepared for students. Amir Mullick -- YouTube.com/USMLELive Best of luck.
S ectIonI I I He mat ol
ogyandonc ol ogy he matol
ogya
ndonc ol
ogy
—PhaRmac ology

Et
opos
ide
,teni
pos
ide There are 2 "sides" to everything; topoisomerase 2. There are 2 testicles and 2 lungs.

me
cha
nIS
m I
nhi
bitt
opo
iso
mer
aseI
I DNAde
gra
dat
ion.
c
lI
nI
calU
Se So
lidt
umo
rs(
par
ti
cul
arl
yte
sti
cul
ara
nds
mal
lce
lll
ungc
anc
er)
,le
uke
mia
s,l
ymphoma
s.
a
dVe
RSee
FFe
ctS My
elos
upp
res
si
on,a
lope
cia
.

I
ri
not
ecan,
topot
ecan I know "I can." Iron (Irin) is related to colon cancer. The "top" goes over (ovary) your lungs.

me
cha
nIS
m I
nhi
bitt
opo
iso
mer
aseIa
ndp
rev
entDNAunwi
ndi
nga
ndr
epl
ica
tio
n.
c
lI
nI
calU
Se Co
lonc
anc
er(
ir
ino
tec
an)
;ov
ari
ana
nds
mal
lce
lll
ungc
anc
ers(
topot
eca
n).
a
dVe
RSee
FFe
ctS Se
ver
emy
elos
upp
res
si
on,
dia
rrhe
a.

Hydr
oxyur
ea
me
cha
nIS
m I
nhi
bit
sri
bonuc
leo
tider
educ
tas
e DNASy
nthe
si S-
s( pha
ses
pec
ific
).
c
lI
nI
calU
Se My
elo
pro
lif
era
tiv
edi
sor
der
s(e
g,CML,
pol
ycy
the
miav
era
),s
ick
lec
ell( 
HbF)
.
a
dVe
RSee
FFe
ctS Se
ver
emy
elo
supp
res
si
on.

Bevac
izumab
me
cha
nIS
m Monoc
lonalant
ibodya
gai
nstVEGF.I
nhi
bit
sang
iog
ene
si Be
s( Va
ciz
uma
binhi
bisBl
t oodVe
sse
l
f
ormat
ion)
.
c
lI
nI
calU
Se So
lidt
umo
rs(
eg,
col
ore
cta
lca
nce
r,r
ena
lce
llc
arc
ino
ma)
,we
tag
e-r
ela
tedma
cul
arde
gene
rat
ion.
a
dVe
RSee
FFe
ctS He
mor
rha
ge,
bloodc
lot
s,a
ndi
mpa
ire
dwoundhe
ali
ng.

Er
lot
ini
b
me
cha
nIS
m EGFRt
yros
inek
ina
sei
nhi
bit
or.
c
lI
nI
calU
Se No
n-s
mal
lce
lll
ungc
anc
er.
a
dVe
RSee
FFe
ctS Ra
sh.

Cet
uxi
mab,
pani
tumumab When you wear a tux, your head, neck, and rectum are visible (no pants). The 'c' in the name is for clavical (to not confuse with rituximab).
me
cha
nIS
m Monoc
lona
lant
ibodi
esa
gai
nstEGFR.
c
lI
nI
calU
Se St
ageI
Vco
lor
ect
alc
anc
er(
wil
d-t
ypeKRAS)
,he
ada
ndne
ckc
anc
er.
a
dVe
RSee
FFe
ctS Ra
sh,
ele
vat
edLFTs
,di
arr
hea
.

I
mat
ini
b,das
ati
nib
me
cha
nIS
m Ty
ros
ineki
nasei
nhi
bit
oro
fbcr-a
bl(
enc
odedbyPhi
lade
lphi
achr
omo
somef
usi
ong
enei
nCML)
a
ndc -
kt(
i c
ommoninGIs t
romalt
umors
).
c
lI
nI
calU
Se CML,GIs
tr
oma
ltumo
rs(
GIST)
.
a
dVe
RSee
FFe
ctS Fl
uidr
ete
nti
on.
Prepared for students. Amir Mullick -- YouTube.com/USMLELive Best of luck.
Hema tol ogyandonc ology hema tol
ogyandonc olog
y—PhaR mac
ology S
ect IonI I
I

Ri
tuxi
mab 'Rit'uximab gets rit (rid) of B cells.

me
cha
nIS
m Mono
clona
lant
ibodya
gai
nstCD2
0,whi
chi
sfoundonmos
tB-
cel
lne
opl
asms
.
c
lI
nI
calU
Se Non-
Hodg
kinl
ymp
homa
,CLL,I
TP,r
heuma
toi
dar
thr
it
is
.
a
dVe
RSee
FFe
ctS r
is
kofpr
ogr
ess
ivemul
ti
foc
all
euk
oenc
epha
lopa
thy
.

Bor
tez
omi
b,c
arfl
zomi
b
me
cha
nIS
m Pr
ote
aso
mei
nhi
bit
ors
,induc
ear
res
tatG2-
M pha
sea
nda
popt
osi
s.
c
lI
nI
calU
Se Mul
ti
plemy
eloma
,ma
ntl
ece
lll
ympho
ma.
a
dVe
RSee
FFe
ctS Pe
riphe
ralne
uro
pat
hy,he
rpe
szos
terr
eac
tiv
ati
on.

T
amox
ifen,
ral
oxi
fene
me
cha
nIS
m Se
lect
iv
ee s
troge
nrece
ptormodula
tor
s(SERMs
)—rece
pto
rant
ago
nis
tsi
nbr
eas
tanda
goni
st
sin
bone
.Blockthebi
ndingofe
str
ogentoER⊕ c
ell
s.
c
lI
nI
calU
Se Br
eastca
ncertr
eat
ment(
tamo
xif
eno
nly
)andpr
eve
nti
on.
Ral
oxi
fenea
lsous
efult
opr
eve
nt
os
teopor
osi
s.
a
dVe
RSee
FFe
ctS Tamoxif
en—parti
alag
onisti
ne ndo metr
ium,whi
ch ther
iskofendomet
ri
alcanc
er;“hotfashes
.”
Ralo
xif
ene—no i nendometri
a lcarci
noma(soy
oucanre
lax!
),b
e c
ausei
tisa
ne s
trog
e nrec
ep t
or
ant
agonis
tinendometr
ialt
is
sue .
Both ris
kofthro
mboe mbol
ice vents(
eg,DVT,PE)
.

T
ras
tuz
umab
me
cha
nIS
m Monocl
onalant
ibodyagai
nstHER-2(
c-
erbB2
),at
yros
inekina
ser
eceptor
.Helpski
llca
ncercel
ls
t
hatov
erexpr
essHER-2throug
hinhi
bit
ionofHER-2ini
ti
ate
dcel
lula
rsigna
lingandant
ibody
-
d
ependentcyt
otox
ici
ty
.
c
lI
nI
calU
Se 2⊕ br
HER- eas
tca
nce
randg
ast
ri
cca
nce
r(t
ra2z
s uma
b).
a
dVe
RSee
FFe
ctS Ca
rdi
oto
xic
ity
.“He
art
cep
tin”da
mag
e hehe
st art
.
"Dont trast HER, she will break your heart."

Dabr
afeni
b,vemur
afeni
b
me
cha
nIS
m Smal
lmole
cul
einhibi
tor
sofBRAFonc
oge
ne⊕ me l
a .VEmuRAF-
noma e
nibisf
orV600E-
mut
atdBRAFi
e nhibi
ti
on.Oft
enc
o-a
dmini
st
ere
dwi t
hMEKinhi
bit
ors(
eg,t
ramet
ini
b).
c
lI
nI
calU
Se Me
tas
tat
icme
lanoma
.

Ras
bur
icas
e
me
cha
nIS
m Re
combi
nantur
ica
set
hatc
ata
lyz
esme
tabol
is
m ofur
ica
cidt
oal
lant
oin.
c
lI
nI
calU
Se Pr
eve
nti
ona
ndt
rea
tme
nto
ftumo
rly
siss
yndr
ome
.

UWORLD= A 46yo woman comes to the clinic to establish primary care as a new patient. She is a marathon runner but has been unable to train
due to a vertebral stress fracture diagnosed via x-ray during a recent emergency department visit. Her last menstrual period was a year ago. Her
mother died of breast cancer at age 52. She is concerned that her family history puts her at risk for breast cancer. X-ray absorptiometry studies
demonstrate abnormally low bone density in the lumbar vertebrae. What drug option is most appropriate for decreasing the risk of both bone
fractures and breast cancer in this patient? Raloxifene. The patient has amenorrhea and low bone density, suggesting a low-estrogen state,
possibly due to menopause. Her exercise history suggests possible hypothalamic amenorrhea (excessive exercise suppresses secretion of GnRH,
leading to low FSH and estrogen). Low estrogen leads to osteoporosis. Estrogen replacement therapy may mitigate this risk, but it can increase
the risk of breast cancer and should be avoided in patients with a family history of breast cancer. Furthermore, unopposed estrogen can cause
endometrial proliferation resulting in endometrial hyperplasia and cancer. Raloxifene and tamoxifen are nonsteroidal compounds that bind
estrogen receptors and exhibit antagonist and agonist properties in a tissue-specific manner. Raloxifene has estrogen agonist activity on bone,
which decreases bone resorption, improves bone density, and decreases the risk of vertebral fractures. In addition, raloxifene has estrogen
antagonist effect on breast tissue and can decrease the risk of breast cancer. It also acts as an estrogen antagonist in the uterus and does not
increase the risk of endometrial cancer.
Prepared for students. Amir Mullick -- YouTube.com/USMLELive Best of luck.
S ectIonI I I He mat ol
ogyandonc ol ogy he matol
ogya
ndonc ol
ogy
—PhaRmac ology

Keyc
hemot
oxi
ci
ti
es
Ci
spl
at
in
/Ca
rbo
pla
ti
n o
tot
oxi
ci
ty

Vi
ncr
is
ti
ne p eri
pher
aln
europa
thy
Bl
eomyci
n,Bus
ulf
an p ul
mon a
ryfibr
osi
s
Dox
orubi
ci
n c a
rdi
ot
oxi
ci
ty
Tr
ast
uzumab c ar
dio
tox
ici
t
y
Ci
spl
at
inCa
/ rb
opla
ti
n n ephrot
oxi
ci
ty

CY
clop
hos
pha
mid
e h
emo
rrh
agi
ccy
sti
t
is

You might also like